MBE KAPLAN--CRIM PRO Flashcards

1
Q
  1. A defendant was employed by a railroad company as a watchman at its crossing, to give warning to the public of approaching trains. Late one evening, he fell asleep in his kiosk and failed to warn of the approach of an oncoming train. A man, who was driving his car, knew of the usual presence of the watchman. As he approached the crossing, he received no warning. Driving onto the track, the man’s car was struck and crushed by the train. The man died instantly.
    A few minutes after the accident, a police officer arrived at the crossing and walked into the kiosk. As he entered, the defendant then awoke. The police officer asked him, “What happened here?” The defendant made incriminating statements.
    The defendant is subsequently charged with involuntary manslaughter. His motion to prevent the introduction of his incriminating statements into evidence will most likely be
    (A) granted, because the police officer failed to give the defendant his Miranda warnings.
    (B) granted, because the police officer’s conduct in questioning the defendant immediately after he awoke was unfairly prejudicial to the defendant.
    (C) denied, because the exchange took place in a non-custodial setting and the question was investigatory in nature.
    (D) denied, because the defendant’s incriminating statements were voluntary.
A
  1. (C) With Miranda, the Fifth Amendment privilege against compelled self-incrimination became the basis for ruling upon the admissibility of a confession. Miranda warnings (and a valid waiver) are prerequisites to the admissibility of any statement made by the accused during a custodial interrogation. Generally speaking, an interrogation will be considered custodial if the individual is not free to leave. According to Orozco v. Texas, 394 u.s. 324 (1969), the Court held that a person is in “custody” if he is not free to leave, even if the questioning occurs in the individual’s bedroom. In the present example, the facts do not indicate that the defendant was in police “custody” for Miranda purposes. Therefore, choice (C) is the best answer because the defendant’s statements were not made during a custodial interrogation. Choice (C) is a better answer than choice (D) because the issue here is whether the police questioning was “on-the-scene” or “custodial.” Choices (A) and (B) are incorrect for the reasons stated above.
How well did you know this?
1
Not at all
2
3
4
5
Perfectly
2
Q
  1. A defendant was convicted of assault of a federal officer and imprisoned in a federal penitentiary. While in prison, federal authorities began investigating the murder of the defendant’s 11-year- old stepdaughter who was murdered two years earlier. Believing that the defendant was responsible for this killing, federal agents decided to plant an informant at the prison to befriend the defendant and possibly obtain evidence about the crime.
    In accord with this plan, the federal agents hired a paid informant and placed him in the prison as a fellow inmate, masquerading as an organized crime figure. The informant soon befriended the defendant and learned that he was being threatened by other inmates because of a rumor that he had killed a child. The informant then raised the subject in several conversations, but the defendant repeatedly denied any such involvement. After the defendant started receiving tough treatment from other inmates because of the rumor, the informant offered to protect the defendant but told him, “You have to tell me the truth about what really happened you know, if you want me to help you.” Believing that the informant would protect him from the other inmates, the defendant then admitted to the informant that he had driven his stepdaughter to the desert where he killed her.
    Based upon this confession, the defendant was then indicted for the first-degree murder of his stepdaughter. Prior to trial, the defendant moves to suppress the statement given to the informant.
    Should the defendant’s motion be granted?
    (A) Yes, because the confession was coerced by the threat of physical violence, absent protection from the informant that motivated the defendant to confess.
    (B) Yes, because the informant was a false friend, and the defendant was tricked into making the confession.
    (C) No, because the confession was voluntary.
    (D) No, because under the totality of circumstances, the informant’s conduct was not inherently coercive.
A
  1. (A) Under the voluntariness standard, the issue of whether a defendant’s confession will be admissible is determined by a “totality of circumstances” approach, which examines both (1) the nature of the defendant (i.e., age, sex, race, mental condition, physical condition, history of drug or alcohol abuse), and (2) the nature of the police conduct. These factors help determine the extent to which the defendant’s ability to submit to external pressures has been affected. Confessions obtained under conditions where the defendant’s free choice is significantly impaired are likely to be found coercive. However, where the police employ a “false friend,” such as a jail cell “plant,” and by deception the defendant is unaware that the person with whom he is conversing is a police officer or agent, a confession thereby obtained will not necessarily be involuntary, even if the defendant mistakenly believed the person could be trusted. LaFave, Criminal Procedure. Choice (B) is incorrect. Nevertheless, where actual or threatened physical harm or brutality is involved, the Court has readily found there to be coercion sufficient to negate the defendant’s free will. In Arizona v. Fulminante, 59 LW 4235 (1991) under similar facts regarding the murder of the defendant’s stepdaughter, the Court heLd that a credible threat of physical violence is sufficient to support a finding of coercion. By intimidating the defendant with the threat of physical violence from other inmates unless he sought the informant’s protection, the government employed coercive tactics in obtaining the defendant’s confession. Therefore, choice (A) is correct. Choices (C) and (D) are incorrect for the reasons stated above.
How well did you know this?
1
Not at all
2
3
4
5
Perfectly
3
Q
  1. Late one evening, a police department received a telephone call from an unidentified woman who reported hearing loud shrieks and screams from a neighboring home. The caller furnished the police with the address where the disturbance was taking place. When the police arrived at the home, they discovered the homeowner, bludgeoned to death. The murder weapon was a blood-stained baseball bat found near the victim’s body.
    A subsequent investigation revealed that the homeowner had recently been separated from her husband who had since moved to another city. After questioning several of the victim’s friends and relatives, the police concluded that the husband was a prime suspect in the murder. Thus, two detectives went to question the husband about the killing. When they arrived at his apartment, the detectives persuaded the landlord to let them into his apartment. Finding no one inside, they searched the apartment. Before leaving, the detectives took a box of cereal, which they planned to use for a fingerprint comparison. The prints from the cereal box matched those found on the baseball bat. The police provided the grand jury investigating the murder with the fingerprint comparison.
    The husband is subsequently subpoenaed to testify before the grand jury. Before his scheduled appearance, the husband files a motion to suppress evidence of the fingerprint comparison, contending that the evidence was illegally obtained.
    His motion should be
    (A) granted, because the warrantless search of the husband’s apartment was unconstitutional.
    (B) granted, because the grand jury is barred from considering illegally obtained evidence.
    (C) denied, because the exclusionary rule has not been extended to grand jury hearings.
    (D) denied, because the landlord had the apparent authority to authorize the search of the husband’s apartment.
A
  1. (C) In United States v. Calandra, 414 U.S. 338 (1974), the Court refused to extend the exclusionary rule to grand jury proceedings. A divided Supreme Court (6-3) noted that “in deciding whether to extend the exclusionary rule to grand jury proceedings, we must weigh the potential injury to the historic role and functions of the grand jury against the potential benefits of the rule as applied in this context. It is evident that this extension of the exclusionary rule would seriously impede the grand jury.” Therefore, choices (A), (B), and (D) are incorrect.
How well did you know this?
1
Not at all
2
3
4
5
Perfectly
4
Q
  1. A defendant was suspected of having burglarized his neighbor’s apartment. The neighbor reported that his apartment had been ransacked and several items of clothing had been stolen. During the course of their investigation, two police detectives went to the defendant’s place of work to interview him. After being advised of his Miranda rights, the defendant requested permission to call his attorney. Although his attorney was unavailable, the attorney’s receptionist admonished him not to say anything. The defendant told the detectives he would have nothing further to say unless his attorney was present. The detectives then asked him if he would accompany them to the police station to answer some questions about an unrelated robbery. The defendant agreed.
    As they were driving to the police station, the defendant was asked where he had purchased the boots that he was wearing. He refused to answer. When they arrived at the police station, the detectives requested that he sign a waiver of his right to counsel. The defendant replied that he would not sign anything in the absence of his attorney. He was then placed in an interrogation room. Shortly thereafter, the defendant was told he could leave if he would be willing to say where he had purchased his boots. The defendant admitted that he had bought the boots and some shirts from a friend. From this information, the detectives obtained a search warrant and went to the defendant’s home where they found the stolen clothing.
    The defendant is charged with burglarizing the apartment. At trial, the court should
    (A) admit the confession because it was voluntary, and the clothing because it was obtained pursuant to a valid search warrant.
    (B) suppress the confession because it was obtained in violation of his Sixth Amendment right to counsel, but admit the clothing because it was obtained pursuant to a valid search warrant.
    (C) suppress the confession because the defendant failed to sign the waiver, but admit the clothing because it was obtained pursuant to a valid search warrant.
    (D) suppress the confession because it was obtained in violation of his Fifth Amendment right to counsel, and the clothing because the search warrant was secured as a result of the confession.
A
  1. (D) In Miranda, the U.S. Supreme Court said “if the individual states that he wants an attorney, the interrogation must cease until an attorney is present.” In addition, the Court stated that “if the interrogation continues without the presence of an attorney and a statement is taken, a heavy burden rests on the government to demonstrate that the defendant knowingly and intelligentLy waived his privilege against self-incrimination and his right to retained or appointed counsel.”
How well did you know this?
1
Not at all
2
3
4
5
Perfectly
5
Q
  1. A student was a junior criminology major at state university and a member of a fraternity. While new members were pledging the fraternity, the student had a reputation for initiating pranks and hazing the pledges. Late one night, after a fraternity party, the student decided to kidnap one of the pledges. Enlisting the help of his friend, they grabbed the pledge, tied him up, and locked him in the trunk of the student’s car. They then drove into the downtown section of a city where they dropped the pledge at a street corner. While the pledge was wandering around and trying to find a telephone booth, a gang of youths saw his predicament and attacked him. The pledge, who was severely beaten, suffered a broken nose, multiple lacerations, and contusions.
    When the pledge finally returned to the school, he filed a criminal complaint against the student who was charged with violating a state law, which provides:
    “Every person who hazes a student and thereby is responsible for causing bodily harm to said student is guilty of a felony punishable by three years in prison.”
    At thai, the prosecuting attorney called the student’s alleged accomplice as a witness. The friend refused to answer any questions and was cited for contempt. After the friend left the witness stand, the prosecutor offered into evidence a transcript of the friend’s testimony given at the student’s preliminary hearing. At the preliminary hearing, the friend testified under oath that he and the student were responsible for hazing the pledge and driving the victim against his will to the city. During the preliminary hearing, the friend was also cross-examined by the student’s defense counsel. Over defense objections, the trial court admitted the transcript of the friend’s testimony at the preliminary hearing. Thereafter, the student was convicted of violating the aforementioned hazing statute.
    The student appeals the conviction and contends that the admission of the transcript of the friend’s testimony at the preliminary hearing violated his Sixth Amendment right of confrontation.
    As to this claim, the student’s appeal will most likely be
    (A) granted, because there was no opportunity to cross-examine the witness at trial.
    (B) granted, because in order to admit prior testimony, the witness must be shown to be unavailable.
    (C) denied, because the witness was unavailable, and there was adequate opportunity for crossexamination at the preliminary hearing.
    (D) denied, because the testimony was a statement by a co-conspirator and, therefore, admissible as a recognized exception to the hearsay rule.
A
  1. (C) Former testimony under FRE 804(b)(1) is defined as “testimony given as a witness at another hearing of the same or a different proceeding, or in a deposition taken in compliance with law in the course of the same or another proceeding, if the party against whom the testimony is now offered, or, in a civil action or proceeding, a predecessor in interest, had an opportunity and similar motive to develop the testimony by direct, cross, or redirect examination.” The transcript of the friend’s testimony at the preliminary hearing will be admissible as former testimony, and the student’s cLaim will be denied. Similarly, in California v. Green, 399 U.S. 149 (1969), the use of preliminary hearing testimony of a witness who was unavailable at trial was held not to violate the defendant’s constitutionaL right of confrontation. Choice (C) is thus correct. Choice (A) is incorrect because the opportunity to cross- examine the friend at trial is unnecessary, since he was already cross-examined by the student’s attorney at the preliminary hearing concerning the testimony given at that time. Choice (B) is incorrect because under FRE 804(a)(2), refusal to testify constitutes unavailability. Choice (D) is incorrect because it mixes two different rules. First, under FRE 804(b) (3), a declaration against interest must be against the declarant’s (penal) interest at the time when made. The friend’s statement incriminated the student as the perpetrator, not himself. Second, a co-conspirator’s admission under FRE 801 (d)(2)(e) is defined as non-hearsay—not as a hearsay exception—under the FRE.
How well did you know this?
1
Not at all
2
3
4
5
Perfectly
6
Q
  1. In her request for jury instructions on the defense of duress, a defendant argued that she should have the burden of producing some evidence of duress, but the prosecution should be required to disprove duress beyond a reasonable doubt. The trial court denied the defendant’s request and instead instructed the jury that the defendant had the burden of proof to establish the defense of duress by a preponderance of the evidence. After she was convicted by the jury, the defendant claims on appeal that the jury instructions regarding duress violated the due process clause.
    Were the jury instructions correct?
    (A) No, because the prosecution bears the burden of proof beyond a reasonable doubt on all elements and defenses produced at trial by a defendant.
    (B) No, because the jury instruction impermissibly shifts the burden of proof to the defendant.
    (C) No, because the defense of duress is no longer a recognized defense in federal prosecutions, although it was recognized at common law, and is recognized in most state courts.
    (D) Yes, because the defense has the burden of proving the defense of duress by a preponderance of the evidence.
A
  1. (D) In Dixon v. United States, 126 S. Ct. 2437 (2006), the Supreme Court ruled that, in the absence of a federal statute, modern common law does not require the prosecution to bear the burden of disproving defendant’s duress beyond a reasonable doubt. Instead, the Court held that Congress intended the defendant to bear the burden of proving the duress defense by a preponderance of the evidence. Note:
    Congress can change the result in Dixon by statute. Choices (A), (B), and (C) are incorrect for the reasons stated above.
How well did you know this?
1
Not at all
2
3
4
5
Perfectly
7
Q
  1. Under which of the following fact situations would the defendant’s Miranda waiver most likely be ineffective?
    (A) A defendant recently graduated from law school. At her graduation party, the defendant became highly intoxicated after drinking a pint of whiskey. Following the party, the defendant attempted to drive home in her car. She fell asleep at the wheel and crashed into another vehicle, seriously injuring the driver. Shortly after the accident, a police officer came on the scene and arrested the defendant, charging her with D.U.I. The defendant was then given her Miranda warnings and transported to the police station. Upon questioning, the defendant, who was still highly intoxicated, waived her Miranda rights, and the police elicited an incriminating statement.
    (B) A defendant stabbed a victim after a violent argument. Following the stabbing death, the police arrested the defendant and charged him with murder. He was transported to the station house where Miranda warnings were given. Afterward, the defendant was interrogated and proceeded to waive his Miranda rights. He then confessed to committing the crime. At trial, a psychiatrist testified that the defendant was mentally ill and his confession was not the result of a knowing and intelligent waiver.
    (C) A defendant was a 15-year-old boy who was a high school sophomore. He possessed normal intelligence and experience for a youth of his age. One night he and two friends attended a concert in the park. After the concert, the defendant and his friends went on a spree, assaulting and robbing a number of victims in the park. The next day, the defendant was arrested. After being subjected to persistent questioning for two hours, the defendant waived his Miranda rights and made a confession. At trial, the defendant claims that he did not make a knowing and intelligent waiver.
    (D) A defendant was a 16-year-old juvenile who was in police custody on suspicion of murder. He was given his Miranda warnings and he then requested to have his probation officer present. He had been on probation for a series of juvenile offenses. His request was denied. During a brief interrogation, the defendant proceeded to waive his Miranda rights and made incriminating statements that linked him with the crime. At trial, the defendant’s lawyer claims that his waiver was ineffective because his request to see the probation officer was the equivalent of asking for a lawyer.
A
  1. (C) The Miranda right to silence can be waived either expressly or impLiedly. Miranda waiver is based on voluntariness as determined by the “totality of circumstances.” To determine if a knowing and intelLigent waiver has occurred, the Court views both the (1) competence of the defendant (i.e., age, experience, inteLligence, and ability to fully understand the warnings), and (2) the conduct of the police, namely as to whether there has been overreaching. Choice (A) is wrong because defendants have generally been unsuccessful in claiming that their Miranda waivers should be held invalid because they were either intoxicated or under the influence of drugs or medication at that time. Likewise, choice (B) is wrong inasmuch as the “personal characteristics of the defendant existing at the time of the purported waiver are relevant only as they relate to police overreaching.” See Colorado v. Connelly, 479 U.S. 157 (1986), where the Court rejected a state court ruling that a defendant’s Miranda waiver was not voluntary because he suffered from a psychosis that interfered with his ability to make free and rational choices. The Court concluded that “Miranda protects defendants against government coercion but goes no further than that.” As a result, choice (C) is the best answer because the defendant was subjected to persistent questioning for two hours before waiving his Miranda rights. According to LaFave, courts have held waivers invalid where the defendant had been held in custody for an extended period of time before being given the warnings, orwhere the defendant had first been subjected to persistent questioning. Lastly, choice (D) is incorrect because in Fare v. Michael C., 442 U.S. 707 (1979), the Court held that a juvenile’s request to have his probation officer present was notaperse invocation of Mfranda rights.
How well did you know this?
1
Not at all
2
3
4
5
Perfectly
8
Q
  1. A defendant was arrested and charged with conspiracy to receive stolen property. At his arraignment, the defendant was represented by counsel. He was then released after posting bond. Following his release, the defendant resumed his job as a bartender at a local bar. Three weeks before the defendant’s scheduled trial, an informant entered the bar. After a few drinks, the informant began conversing with the defendant, who was on duty at the time. Unknown to the defendant, the man was a paid police informant. During the course of their conversation, the informant told the defendant that he had read about his arrest in the newspapers and questioned the defendant about the names of his accomplices. Unsuspectingly, the defendant made some admissions, which the informant then passed on to the prosecuting attorney.
    At trial, the prosecution tried to introduce into evidence the defendant’s admissions. The defendant’s motion to exclude this offer of proof will most likely be
    (A) denied, because the defendant’s statements were voluntary.
    (B) denied, because the defendant assumed the risk that his confidence in the informant was not misplaced.
    (C) granted, because the defendant’s Sixth Amendment right to counsel was violated.
    (D) granted, because the defendant’s Fifth Amendment privilege against self-incrimination was violated.
A
  1. (C) The clear rule in Massiah v. United States, 377 U.s. 201 (1964), is that once adversary proceedings have been commenced against an individual, he has a right to legal representation when the government interrogates him. As in Brewer v. WI!hams, 430 U.S. 387 (1977), the critical issue in this example is whether, after judicial proceedings have been initiated against the defendant, a police informant elicited information from him in the absence of defense counsel. According to Brewer, proof of formal interrogation is unnecessary to invoke the protection of the Sixth Amendment. A conversation that is tantamount to interrogation is sufficient. Similarly, since the defendant was under arraignment, the informant could not effectively “interrogate” him (i.e., attempt to obtain information reasonably calculated to induce conversations relative to the crime) in the absence of defense counsel. Therefore choices (A) and (B) are incorrect. Choice (D) is incorrect. Due to the fact that we have reached a critical stage, namely the arraignment, it is the Sixth Amendment right to counsel that has attached.
How well did you know this?
1
Not at all
2
3
4
5
Perfectly
9
Q
  1. One evening, a defendant set fire to an occupied house. As a result of the blaze, the homeowner’s daughter was killed. The defendant was charged with felony murder on the first count and arson on the second count of the two-count indictment. The jury found the defendant guilty on the first count, but returned a not guilty verdict on the second count.
    The defendant’s attorney’s motion to set aside the guilty verdict on the felony murder charge will be
    (A) granted, because the guilty verdict is plain error that adversely affects the defendant’s constitutional rights.
    (B) granted, because the verdicts are legally inconsistent and should lead to an acquittal of both charges.
    (C) denied, because the verdicts do not amount to a reversible error.
    (D) denied, because the defendant’s proper remedy is to seek an appellate review for a non-constitutional error.
A
  1. (B) In order for one to be found guilty of murder under the felony murder rule, he must also be found guilty of the underlying felony. Thus, as in the present example, if a defendant is found innocent of the underlying felony, be cannot be found guilty of felony murder. Briefly, the felony murder rule provides that one whose conduct brought about an unintended death in the commission or attempted commission of an inherently dangerous felony was guilty of murder. Choices (A), (C), and (D) are incorrect for the reasons stated above.
How well did you know this?
1
Not at all
2
3
4
5
Perfectly
10
Q
  1. Police received reliable information that a homeowner had a stolen x-brand stereo in his possession. The detectives then submitted an affidavit to a neutral magistrate setting forth sufficient underlying circumstances for the issuance of a search warrant. Making a determination of probable cause, the magistrate issued a warrant for the x-brand stereo at the homeowner’s address.
    The police arrived at the homeowner’s dwelling, showed him the warrant, and came inside. In the living room, they noticed a y-brand stereo, which had the serial number removed. Upon further inspection, the police determined that the stereo had been stolen from a local electronics store during a recent burglary. Thereupon, the police placed the homeowner under arrest and instructed him to remain seated in the living room while they searched the rest of the home. One of the officers proceeded to the basement where she found the stolen x-brand stereo. She then decided to search the upstairs and came upon a stolen z-brand stereo in the homeowner’s second-floor bedroom.
    The homeowner was subsequently prosecuted for receiving stolen property. At trial, the homeowner moves to prevent introduction of the stereos into evidence. His motion should be granted with respect to
    (A) the y-brand stereo.
    (B) the z-brand stereo.
    (C) the y-brand and the z-brand stereos.
    (D) none of the stereos.
A
  1. (C) A search made under authority of a search warrant may extend to the entire area covered by the warrant’s description. For example, if the warrant authorizes a search of “premises” at a certain described geographical location, buildings standing on that land may be searched. If the place is identified by a street number, the search may extend to those buildings within the curtilage and the yard within the curtilage. LaFave points out that the permissible intensity of the search within the described premises is determined by the description of the things to be seized.
    Here, the warrant covered the seizure of a stolen x-brand stereo at the homeowner’s address. When the police arrived at the defendant’s home, they noticed a y-brand stereo in the living room. The first question is whether the police, under the “plain view” doctrine, were permitted to seize the y-brand stereo, which also turned out to be stolen. The “plain view” doctrine is legitimate only where it is immediately apparent to the police that they have evidence before them; the “plain view” doctrine may not be used to extend a general exploratory search from one object to another. In fact, in Arizona v. Hicks, 480 U.S. 321 (1987), it was held that full probable cause was needed to pick up an item of stereo equipment to ascertain its serial number (which revealed it was stolen). Based on Hicks, the y-brand stereo was unlawfully seized since the police needed to “further inspect” it to determine its stolen status. By the same token, the police did not have authority to continue to search the home after discovering the stolen x-brand stereo (specified in the warrant). When the purpose(s) of the warrant have been carried out, the authority to search is atan end. Choice (C) is correct because the defendant’s motion to exclude the y-brand and the z-brand stereos will be granted. Therefore, (A), (B), and (D) are incorrect.
How well did you know this?
1
Not at all
2
3
4
5
Perfectly
11
Q
  1. A defendant was prosecuted for murder. At trial, the prosecutor called a police detective to testif’. The detective, who questioned the defendant at the station house after his arrest, testified that the defendant initially declared his innocence. The detective further testified that when given the opportunity to take a lie detector test, the defendant refused. The defendant’s attorney did not object to the detective’s testimony. The defendant was convicted and sentenced to prison.
    The defendant, who is now represented by a new attorney, appeals, claiming that it was error to admit the detective’s testimony. The appellate court should find
    (A) plain error, because the trial court should have acted on its own motion to order reference to the lie detector test stricken.
    (B) plain error, because admission of the detective’s testimony violated the defendant’s privilege against self-incrimination.
    (C) harmless error, because the defendant’s trial attorney failed to preserve the argument by timely objection.
    (D) harmless error, because the defendant’s statements were admissions.
A
  1. (A) A distinction frequently tested on the MBE is between “harmless error” and “plain error.” If an error has not resulted in damage to the complaining party, it may be deemed harmless, and a new trial need not be had. For an example of “harmless error,” see United States v. Shepard, 538 R2d 107 (1976) (permitting psychiatrist who examined defendant to determine competency to stand trial and to testify about alibi related to him by defendant). On the other hand, even where no timely objection is made, if a grave injustice might result from a serious trial error, the Appellate Court may still order a new trial. For example, in U.S. v. Sisto, 534 F.2d 616 (1976), it was held to be “plain error” where the judge failed to instruct the jury that an undercover agent’s statements (concerning what an alleged accomplice said about the defendant’s activities) should be considered only as impeachment evidence and not for their truth. The facts of this question raise a general rule concerning criminal prosecutions, which is that evidence is not admissible that the defendant has been willing or unwilling to take a lie detector test. It has been noted that the impact upon the minds of the jurors of a refusal to submit to something which they might well assume would effectively determine guilt or innocence might well be more devastating than a disclosure of the results of such test. In other words, the contention that evidence of the defendant’s refusal to take a lie detector test tends to establish consciousness of guilt, and that evidence of the defendant’s willingness to take such a test shows consciousness of innocence, has been rejected. Choice (A) is a better answer than choice (B) because it addresses the issue that “plain error” affects substantial rights to such a serious degree that a new trial can be given (even though the defendant’s attorney did not object to the detective’s testimony). Choices (C) and (D) are incorrect for the reasons stated above.
How well did you know this?
1
Not at all
2
3
4
5
Perfectly
12
Q
  1. A husband is charged with murder in the shooting death of his brother-in-law. In the jurisdiction in which the husband is on trial for murder, the defense of insanity is an affirmative defense. The husband pleads insanity as his defense.
    At trial, in order to prove his defense of insanity, the husband has
    (A) the burden of production.
    (B) the burden of persuasion only.
    (C) both the burden of production and the burden of persuasion.
    (D) neither the burden of production nor the burden of persuasion.
A
  1. (A) The husband has the burden of production of proving his insanity at the time of the offense. On the issue of lack of responsibility because of insanity, the initial burden of going foiward (the production burden) is placed upon the defendant in every jurisdiction in the United States. The burden of persuasion, on the other hand, after the issue of insanity has been raised, is upon the prosecution. LaFave in Criminal Law notes that the defendant’s production burden is often stated in terms of a presumption of sanity; most men are sane, and thus the defendant in the particular case is presumed to be sane until some amount of evidence to the contrary is produced. Note that the Model Penal Code takes the view as to the affirmative defenses that the accused has the first burden of producing evidence. Choices (B), (C), and (D) are incorrect for the reasons stated above.
How well did you know this?
1
Not at all
2
3
4
5
Perfectly
13
Q
  1. Defendant was on trial for robbery. Defendant’s direct examination continued until late in the day when it finally concluded. At this time, the trial court judge adjourned the proceedings for the evening. The judge then instructed Defendant not to speak with anyone during the night and scheduled cross- examination to begin in the morning.
    The judge’s instruction to Defendant was
    (A) proper, because a judge has broad discretion to instruct witnesses in such a manner.
    (B) proper, because it would have the same effect as permitting cross-examination to continue after direct was concluded.
    (C) improper, because it violates the defendant’s Sixth Amendment right to counsel.
    (D) improper, because it violates the attorney-client privilege.
A
  1. (C) In Geders v. United States, 425 U.S. 80 (1976), the trial court ordered the defendant not to consult his attorney during an overnight recess that separated the direct examination and the cross-examination of the defendant. The court of appeals affirmed the conviction because the defendant made no claim of prejudice from the order. The Supreme Court reversed, holding that the 17-hour denial of counsel, regardless of demonstrated evidence, constituted a deprivation of the effective assistance of counsel. LaFave and Israel, Criminal Procedure. This “evidence” question is obviously very difficult, since it involves knowledge of a specific “criminal procedure” case. In light of Geders, choice (C) is correct, since the judge’s instruction to the defendant denied him his Sixth Amendment right to counsel. Choices (A), (B), and (D) are, therefore, incorrect.
How well did you know this?
1
Not at all
2
3
4
5
Perfectly
14
Q
  1. A defendant was arrested one morning by a police officer for the attempted murder of the victim. The defendant had allegedly fired three shots at the victim, a physical education teacher, in the schoolyard of an elementary school.
    Immediately after the arrest, the arresting officer advised the defendant of his Miranda rights. The defendant responded that he would not make any statement until he consulted his attorney.
    Within minutes, a patrol car arrived and the defendant was taken into the car to be transported to the police station. The arresting officer sat in the front, next to the driver, and the defendant sat alone in the back seat, with his hands cuffed.
    On the way to the station, the driver stated to the arresting officer, “I hope that the gun involved in this crime doesn’t get into the hands of those small children, because one of them could be seriously injured, to say the least.” The defendant interrupted the officers and told them where to look in the schoolyard for the abandoned gun. As a result of this information, the police found the gun where the defendant said it was.
    What is the state’s best rebuttal to the defendant’s argument that the gun was illegally seized?
    (A) The defendant was not entitled to be re-warned of his Miranda rights in the patrol car.
    (B) The defendant was not interrogated in the patrol car.
    (C) The defendant waived his right to consult counsel.
    (D) The seizure of the gun was not the fruit of the defendant’s statement.
A
  1. (B) The state’s best rebuttal to the defendant’s argument of illegal seizure of the gun would be that the defendant was not being interrogated in the police car when he revealed the location of the gun to the police officers. The U.S. Supreme Court in Rhode Island v. Innis, 446 U.S. 21 (1980), held that where a suspect in crimes committed with a shotgun told police of the gun’s location after hearing police talk of a handicapped child’s possibility of finding a gun (when the suspect was being transported to a police station), it was not an interrogation in violation of the suspect’s Miranda rights. In the Innis case, as in our factual presentation, the suspect, upon arrest, was advised of his Miranda rights and refused to make any statements without an attorney present. Furthermore, the suspect was questioned or interrogated by the police officers during the ride to the police station; the suspect acted voluntarily when he heard the police officers mention the possibility of children being injured by the gun. The Supreme Court has held that “interrogation” under Miranda refers not only to express questioning, but also to any words or actions on the part of the police that the police should know or should have known are reasonably likely to elicit an incriminating response from the suspect. Nothing suggests that the police were aware that the defendant was particularly susceptible to an appeal to his conscience concerning the safety of small children. Choice (A) is incorrect because there was no need for the defendant to be re-warned of his Miranda rights because there was no interrogation. Choice (C) is incorrect because the defendant invoked his right to counsel. Choice (D) is incorrect because the gun was the fruit of the statement, but no interrogation took place, so Miranda was not violated.
How well did you know this?
1
Not at all
2
3
4
5
Perfectly
15
Q
  1. A town had been experiencing a rash of bank robberies. All the witnesses at each bank had described a man with a gun who said, “Get down on the floor and no one has to die today!” Following a series of leads, the detectives in the case finally made an arrest.
    At the police station, the suspect was placed in a lineup. One of the victims observed the lineup, and after each of the six participants stated, “Get down on the floor and no one has to die today,” the victim identified the suspect as the perpetrator. The defendant’s attorney subsequently moved to suppress the out-of-court identification, arguing that because the defendant was forced to speak, this violated his Fifth Amendment right to remain silent, which had been invoked previously by the defendant.
    The police requirement that the defendant state at the lineup, “Get down on the floor and no one has to die today”
    (A) invalidated the lineup.
    (B) violated the defendant’s right against self-incrimination.
    (C) tainted the victim’s identification of the defendant.
    (D) would not prohibit the defendant’s lineup identification from being introduced at trial.
A
  1. (D) The defendant’s statement during the lineup would not be testimonial in nature. Therefore, the victim’s lineup identification may be introduced by the prosecution at trial. Students should note that appearing in a lineup and being required to make statements during the lineup procedure are not testimonial activities, but demonstrative in nature. Thus, a suspect does not have a right under the Fifth Amendment privilege against self-incrimination to refuse to appear or make a statement during a lineup. See United States v. Wade, 388 U.S. 218 (1967), where the Court held that an accused’s privilege against self-incrimination was not violated by the lineup itself or by requiring the accused to speak the words allegedly uttered by the robber. Choice (A) is incorrect for reasons stated above. Choice (B) is incorrect because, as stated above, there is no Fifth Amendment privilege here. Choice (C) is incorrect because the defendant’s statement did not taint any identification by the victim. Typically, lineups are tainted by improper suggestion by police hints, or unduly suggestive lineups, which tend to point the finger at one particular suspect. For example, if a white man is described as the suspect and the lineup contains one white man and five Hispanic or African American men, then that would be considered unduly suggestive.
How well did you know this?
1
Not at all
2
3
4
5
Perfectly
16
Q
  1. An older and younger brother had always done everything together. They had grown up together, gone to school together, and partied together. One night, while drinking at a bar, both brothers decided that they wanted a little more action than the bar was offering up, so they left and went to a strip club.
    At the strip club, they paid for private dances and attempted to have conversations with the women. As hard as they tried, the women who worked in the club did not seem as interested in the brothers as they were in the women.
    As the night wore on, the brothers decided that they would wait behind the club until closing for the women to come out. The victim, who had given a dance to one of the brothers, came out and proceeded to walk to her car. One of the brothers grabbed the victim from behind and pulled her to the ground while the other brother sexually assaulted her.
    Both brothers were arrested and charged with the rape of the victim. Their long-time family attorney represented them both at their trial. The younger brother intended to testify that his older brother had forced him to hold the victim down by threatening him with a gun.
    If appropriate objections are made by either brother, the court should rule that the attorney’s representation of the brothers at trial
    (A) violated the younger brother’s Sixth Amendment right to counsel.
    (B) constituted an impermissible conflict of interest.
    (C) violated the younger brother’s constitutional rights, but did not infringe on the older brother’s constitutional rights.
    (D) did not violate the older brother’s constitutional rights if he separately paid for his representation.
A
  1. (A) The Sixth Amendment, which is applicable to the states through the Fourteenth Amendment, guarantees an accused the right to be represented by counsel. The attorney’s dual representation of both brothers created a conflict of interest, which violated the younger brother’s Sixth Amendment rights. The U.S. Supreme Court in Cuylerv. Sullivan, 446 U.S. 335 (1980), held that multiple representation in the sense that the same counsel actively represented two defendants charged with the same crime does not violate the Sixth Amendment unless it gives rise to a conflict of interest. The Court further stated that since a conflict of interest inheres in almost every instance of multiple representation, a defendant who objects to it must have the opportunity to show a potential conflict that imperils his right to a fair trial. The younger brother’s incriminating testimony about the older brother at trial should have been sufficient for the court to inquire as to whether or not a conflict of interest existed over the attorney’s dual representation of the defendants. In other words, the trial court more or less had a duty to inquire as to whether there was a conflict of interest as a result of the younger brother’s defense. Although choice (B) provides a correct statement of fact, choice (A) is the better alternative, since it provides a correct statement of law. Choice (C) is incorrect because it is does not specifically address the issues presented by these facts. Choice (A) is better because it directly addresses the specific constitutional amendment. Choice (D) is incorrect because the manner of payment for the representation does not cure the conflict.
How well did you know this?
1
Not at all
2
3
4
5
Perfectly
17
Q
  1. A riot broke out in a city, and many shops had been looted and some had been burned during the riots. Goods of all kinds were in piles or strewn about the sidewalk. During the riot, a defendant was walking along the street when he paused in front of an appliance store. He picked up a portable television set that was lying on the sidewalk and started to walk off quickly. A police officer who was standing in front of the shop rushed after the defendant and arrested him.
    The defendant, an indigent, was charged with grand larceny, a felony carrying the maximum penalty of seven months imprisonment and a S 1,000 fine. At his arraignment, the judge advised the defendant of his right to have counsel appointed for him. The defendant, however, told the judge that he wanted to represent himself and waive his right to a jury trial. The judge then questioned the defendant at length about his demands. The defendant indicated that he had no previous legal experience. He told the judge that the reason he didn’t want a court-appointed attorney to represent him was because he didn’t trust lawyers. The judge did not find the defendant’s reasons for representing himself to be persuasive and appointed an attorney to represent him.
    Thereafter, an attorney was appointed by the court to represent the defendant. The attorney told the court that the defendant wanted a non-jury trial. However, at the prosecution’s request, the court impaneled a jury. The defendant was subsequently convicted by the jury and sentenced to six months imprisonment. In addition, the defendant was ordered to reimburse the state $450 to cover the cost of his attorney. This reimbursement order was made pursuant to a relevant statute in effect at the time of the defendant’s arrest.
    The defendant appeals both the conviction and the reimbursement order. The appellate court should
    (A) affirm both the conviction and the reimbursement order.
    (B) affirm the conviction, but not the reimbursement order.
    (C) reverse the conviction because he was denied the right to a non-jury trial, but not the reimbursement order.
    (D) reverse both the conviction and the reimbursement order because the defendant was denied the right to represent himself.
A
  1. (D) Here’s another classic Multistate example dealing with waiver of right to counsel. A defendant may waive his constitutional right to assistance of counsel provided he does so “knowingly and intelligently,” which means it must have been the product of a reasoned and deliberate choice based upon adequate knowledge of what the assistance of counsel encompasses. A defendant who acts knowingly and intelligently in waiving his right to counsel has a right to proceed pro Se, since he must be free to decide if counsel will be to his advantage in the case. The defendant had the right to waive representation by counsel. See Faretta v. California, 422 u.s. 806 (1975). Although this right of waiver may be denied if a defendant is not competent to represent himself, there is no evidence that the defendant was incompetent. Therefore, his conviction should be overturned, since he was not given the right to represent himself. In addition, the defendant should not be required to reimburse the state for his attorney’s fees due to the fact that counsel was imposed upon him in violation of his constitutional rights. Note, however, that a state can recover legal costs from an indigent who is convicted and is subsequently able to pay. See Fullerv. Oregon, 417 u.s. 40(1974). Choices (A), (B), and (C) are incorrect for the reasons stated above.
How well did you know this?
1
Not at all
2
3
4
5
Perfectly
18
Q
  1. A state has a hit-and-run statute in effect that makes it a crime to leave the scene of an accident. One afternoon, a 9-year-old victim was riding her bicycle along the street. As the victim swerved into the southbound lane, her bicycle was struck by a car driven by the defendant. The victim was knocked off her bike and thrown onto the sidewalk adjacent to the street. Although the victim received some minor scrapes and bruises, she was not seriously injured. Following the accident, the car driven by the defendant sped away.
    Moments later, however, a tractor-trailer crashed into the rear of a car about 30 feet from where the victim was lying. The car almost instantly caught fire as its gas tank exploded. The victim, who was engulfed in the flaming wreckage, was killed.
    If the defendant is charged with involuntary manslaughter for the death of the victim, the defendant should be found
    (A) guilty, because she unlawfully fled the scene of an accident in violation of her statutory duty.
    (B) guilty, because her failure to render aid to the victim would make the defendant criminally responsible for the victim’s death.
    (C) not guilty, because under the circumstances her failure to aid the victim cannot be a basis for imposing criminal responsibility for the victim’s death.
    (D) not guilty, because there was not a sufficient causal connection between her actions and the victim’s death to impose criminal responsibility.
A
  1. (D) As a general rule, most crimes require the following elements: (1) an act, (2) mental fault (or “guilty mind”), (3) concurrence (or act + mental state), (4) harm, and (5) causation. With crimes so defined as to require not merely conduct but also a specified result of conduct, the defendant’s conduct must be the “legal” or “proximate” cause of the result. The fact that the defendant fled the scene of the accident (in violation of the hit-and-run statute) was not the “legal” cause of the victim’s death. The victim’s death resulted from the car’s gas tank exploding, which was caused by the collision with the tractor-trailer. Although choice (C) is also conceivably correct, choice (D) is preferred because it refers to the requirement of a causal connection, which is a material element in criminal law. Choices (A) and (B) both contain the incorrect result because the defendant was not the legal cause of the victim’s death.
How well did you know this?
1
Not at all
2
3
4
5
Perfectly
19
Q
  1. A state has the following hit-and-run statute in effect:
    “Any driver of a motor vehicle (including but not limited to automobiles, trucks, buses, or motorcycles) involved in an accident or collision resulting in injury or death to a human being shall immediately stop his or her vehicle at the scene of such accident or collision, render necessary aid to the injured victim, and furnish the police or other person(s) at the scene with his or her name, address and driver’s license. Any violation or noncompliance with said statute shall be punished by imprisonment for not less than three years nor more than seven years.”
    The defendant was involved in an accident that involved injuries to the other driver. The defendant, however, knowing his license to be revoked and afraid of being arrested, fled the scene. However, the victim was able to write down the license number of the defendant’s vehicle, and the defendant was rapidly apprehended and put in jail.
    The defendant is charged with violating the aforementioned statute. He files a motion to dismiss on the grounds that the disclosure requirement of the statute violates his privilege against self-incrimination.
    His motion should be
    (A) granted, because the statute makes no provision for Miranda warnings concerning his right to remain silent.
    (B) granted, because the statute requires him to provide incriminating information that can be used againsthim in a criminal prosecution.
    (C) denied, because the legislative intent in enacting the statute was designed to require disclosure of information to be used primarily in civil litigation.
    (D) denied, because in accordance with public policy considerations, the required disclosures are insufficiently testimonial.
A
  1. (D) The u.s. Supreme Court has held that a statute requiring a motorist involved in an accident to stop and give his name and address did not involve self-incrimination in a constitutional sense. Choice (C) is wrong because violation of the statute results in a (criminal) sentence of imprisonment. Therefore, clearly, the legislative intent was not directed for the disclosure requirement to be used primarily in civil litigation. Choices (A) and (B) are incorrect because the statute need not make any provision for Miranda, as the defendant’s Miranda rights are not implicated by a
    requirement to give information at the scene of an accident.
How well did you know this?
1
Not at all
2
3
4
5
Perfectly
20
Q
  1. One evening, an undercover narcotics agent, with the help of a confidential informant, met with a man. The man took the agent to the home of a drug dealer, who sold the agent four pounds of marijuana. At trial, the drug dealer claimed the defense of entrapment and demanded that the informant’s identity be disclosed and that he be produced. At no time prior to trial did the drug dealer seek to subpoena the man. However, the man was a fugitive from justice the whole time, and no subpoena could have been served. The government claims a right to protect the informant’s identity.
    Should the government’s claim be honored?
    (A) Yes, because an informant has a Fifth Amendment privilege against self-incrimination.
    (B) Yes, because informants would not assist in crime prevention unless they were reasonably certain that their identities would be kept secret.
    (C) No, because under the Fifth Amendment, a defendant has the right to be confronted by witnesses against him.
    (D) No, because under the Sixth Amendment, a defendant has the right to a fair trial.
A
  1. (B) In the present fact situation, the government’s claim should be honored. See McCray v. Illinois, 386 U.S. 300 (1967). Choice (A) is incorrect because an unidentified informant has not been extended a Fifth Amendment privilege against self-incrimination. In the McCray case, the court held that it may, in the exercise of its power to formulate evidentiary rules for federal criminal cases, decline to disclose an informer’s identity. Choices (C) and (D) are incorrect for the reasons stated above.
How well did you know this?
1
Not at all
2
3
4
5
Perfectly
21
Q
  1. A defendant was on the first day of her new secretarial job when her boss called her into his office. The boss directly suggested that if the defendant did not go out on a date with him, she would be fired in one week. Every day during the remainder of the week, the boss approached the defendant with his demand, and the defendant refused to cooperate.
    At the end of the week, when the boss called the defendant into his office and again tried to pressure her to go out on a date with him, the defendant knocked him unconscious with a giant stapler and choked him to death.
    The defendant is tried for murder. In accordance with the following statute, the state relies at trial on the presumption of malice:
    “When the act of killing another is proved, malice aforethought shall be presumed, and the burden shall rest upon the party who committed the killing to show that malice did not exist.”
    If the defendant is convicted of first-degree murder and challenges her conviction on the grounds of the above statute, on appeal she will
    (A) win, because the statute is unconstitutional.
    (B) win, because the statute violates due process.
    (C) lose, because she failed to overcome the presumption.
    (D) lose, because the presumption may be rebutted.
A
  1. (B) The defendant will win on the appeaL of her murder conviction under the state murder statute, since the statute placed the burden on the defendant to prove or disprove the element of malice. The U.S. Supreme Court in In re Winship, 397 U.S. 358 (1910), held that the Due Process Clause protects an accused in a criminal case against conviction except upon proof “beyond a reasonable doubt” of every fact necessary to constitute the crime for which the defendant is charged. In other words, the Court in the Winship case held that proof of a criminal charge beyond a reasonable doubt is constitutionaLLy required. Consequently, the murder statute in our question is unconstitutional, since the burden of proving the various elements of the offense (murder) is on the defendant, and not on the prosecution, as required in all jurisdictions. Choice (A) is incorrect as it does not go far enough. It is not enough to say unconstitutional. The precise constitutional violation is a better answer choice. Choices (C) and (D) are incorrect for the reasons stated above.
How well did you know this?
1
Not at all
2
3
4
5
Perfectly
22
Q
  1. A defendant suspected her long-term boyfriend was cheating on her. On a hunch, she went to her boyfriend’s apartment one afternoon and, using her key, she entered the apartment. Once inside, she found her boyfriend and his new girlfriend in bed together. In a rage, the defendant retrieved the gun from the nightstand and killed her boyfriend and his new girlfriend.
    After the shooting, the defendant left on a two-day trip to the mountains to get the week’s events off her mind. She called her teenage neighbor to take care of her apartment while she was gone and to look after her 4-year-old daughter. That night, after the defendant left, the police came to the apartment. They asked the neighbor if they could search the apartment, and the neighbor gave them permission. The police found in the defendant’s bedroom the gun used to kill her boyfriend.
    At a motion to suppress the gun prior to trial, which of the following facts would the defendant’s attorney be LEAST likely to attempt to prove?
    (A) The defendant gave her neighbor the keys to her apartment.
    (B) The police did not have a search warrant.
    (C) The defendant told her neighbor not to answer the door to anyone.
    (D) The police told the neighbor she would be taken to the police station if she refused permission to search.
A
  1. (A) Choice (B) is incorrect, since the defendant’s attorney would attempt to prove that the search of her apartment was illegal and, thus, violative of the Fourth Am endment’s protection against unreasonable searches and seizures. Choice (C) is incorrect because if the defendant told the neighbor not to answer the door to anyone, the neighbor could not exercise apparent authority and, thus, consent to a search of the defendant’s apartment. Choice (D) is also incorrect, since the defendant’s attorneywould attempt to prove thatthe neighborwas coerced into granting consent for the search, thus deeming it an illegal search and seizure. ConsequentLy, choice (A) is the LEAST likely fact that the defendant’s attorney would attempt to prove at the suppression hearing. It is important to note that consent searches are one of the exceptions of the Fourth Amendment requirement of a search warrant. However, for a consent search to be vaLid, three factors must be considered: (1) the person consenting must have the authority to consent to a search of the premises; (2) the person’s consent to the search must be voluntary; and (3) the police may not exceed their search into areas for which consent to a search has not been given.
How well did you know this?
1
Not at all
2
3
4
5
Perfectly
23
Q
  1. A detective received information from an informant, who had given reliable information many times in the past, that a man was a narcotics dealer. Specifically, the informant said that, two months before, he had visited the man’s apartment with a friend and that on that occasion he saw the man sell his friend some heroin. The detective knew that the informant, the man, and the friend were acquaintances. Thereafter, the detective put all this information into affidavit form, appeared before a magistrate, and secured a search warrant for the man’s apartment. The search turned up a supply of heroin.
    The man’s motion to suppress introduction of the heroin into evidence will most probably be
    (A) granted, because a search warrant cannot validly be issued solely on the basis of an informant’s information.
    (B) granted, because the information supplied to the detective concerned an occurrence too remote in time to justify a finding of probable cause at the time of the search.
    (C) granted, because a search for mere evidence alone is improper and illegal.
    (D) denied, because the informant had proven himself reliable in the past, and the information he gave turned out to be correct.
A
  1. (B) The Fourth Amendment states, in part, “… and no warrants shall issue but on probable cause supported by oath or affirmation …“ Searches conducted pursuant to a warrant must be based on an adequate and reasonable showing of probable cause when a police officer provides information to a neutral and detached magistrate by affidavit or by testimony under oath. However, when an informant uses an affidavit to provide information, the rigid Aguilar—Spinelli test has been used to establish (1) probable cause for issuing the warrant, as well as (2) reliability of the informant. More recently, a “totality of the circumstances” approach has been adopted [Illinois v. Gates, 462 U.S. 213 (1983)] to determine whether there is a “fair probability” or “substantial basis” to conclude that contraband wiLl be found at the particular time and place. The fact that the informant saw the man selL heroin two months before is critical because it is too remote in time to justify a present finding of probable cause. Choice (B) is correct. Choices (A) and (C) are incorrect for the reasons stated above. Choice (D) is incorrect because reliability of the informant without probable cause to search is an insufficient basis to issue a warrant.
How well did you know this?
1
Not at all
2
3
4
5
Perfectly
24
Q
  1. During the murder trial of a defendant, the prosecution presented four witnesses to the brutal slaying of the victim. The evidence pointed to the fact that the defendant beat her about the head and neck with a baseball bat, causing severe injuries to her brain and her ultimate death.
    The prosecution rested, and the defendant presented two witnesses, his brother and his girlfriend, who testified that the defendant was dining at an elegant restaurant on the other side of town at the time of the alleged murder. The defendant presented no other witnesses.
    During his closing argument to the jury, the assistant district attorney called attention to the fact that the prosecution witnesses had no apparent reason to have any bias toward the prosecution or against the defendant. He then noted that the defendant’s witnesses had clear motives to falsify their testimony and favor the defendant. The assistant district attorney added, “If the defendant was on the other side of town, why didn’t he tell us himself? Why didn’t he get on the stand? What was he hiding? Those are questions for you, the jury, to answer.”
    The defendant was convicted of first-degree murder and sentenced to life imprisonment.
    On appeal, his conviction should be
    (A) reversed, because the prosecutor improperly referred to the possible motives or interests of the defense witnesses.
    (B) reversed, because the defendant’s constitutional rights were violated in the assistant district attorney’s closing argument.
    (C) reversed, because the assistant district attorney referred to the defendant’s failure to testify.
    (D) reversed, because the assistant district attorney’s argument violated the defendant’s rights under the Fifth and Fourteenth Amendments.
A
  1. (0) In Griffin v. California, 380 U.S. 609 (1965), the U.S. Supreme Court held that the self-incrimination guarantee of the Fifth Amendment, as applicable to the states under the Fourteenth Amendment, forbids either comment by the prosecution of an accused’s silence or instructions by the court that such silence is evidence of guilt. The closing comments by the prosecutorthat the defendant failed to take the stand would be violative of the defendant’s right against self-incrimination. Choice (A) is incorrect because it is not improper to argue the bias of a defense witness. Choice (B) is incorrect because it is not as specific and on point to the facts as choice (D). Choice (D) is a better answer than choice (C) because a correct statement of law is generally preferred over a correct statement of fact.
How well did you know this?
1
Not at all
2
3
4
5
Perfectly
25
Q
  1. A defendant and his co-conspirator were arrested and charged with robbery and conspiracy to commit robbery. Following their arrest, they were both taken to the police station and given Miranda warnings. They both expressed a desire to remain silent until they could consult an attorney. At the station house, they were booked and placed in separate jail cells.
    Later that day, and before any attorney consultations could take place, a police detective went to the coconspirator’s cell and began interrogating him. The detective told the co-conspirator that if he cooperated in their investigation, the prosecuting attorney would drop charges against him. The co-conspirator then reluctantly confessed and implicated the defendant in the commission of the crimes. The co-conspirator also told the police where the defendant had hidden the stolen property. Based on this information, the police retrieved the stolen property, which included a diamond necklace.
    Later the same day, the police went to the defendant’s jail cell and showed him the diamond necklace that they had recovered. They also told the defendant that the co-conspirator had confessed and implicated him in the perpetration of the crime. Confronted by this evidence, the defendant confessed.
    The defendant was then prosecuted for conspiracy and robbery. At the defendant’s trial, the prosecution sought to introduce into evidence the necklace and the defendant’s confession.
    The defendant’s motion to exclude these offers of proof will be
    (A) denied to both the necklace and the confession.
    (B) denied to the necklace, but granted to the confession.
    (C) granted to the necklace, but denied to the confession.
    (D) granted to both the necklace and the confession.
A
  1. (B) The defendant’s confession will be inadmissible, whereas the necklace will be admitted over the defendant’s motion to exclude. As to the necklace, the rule is that a conspirator does not have automatic standing to challenge the seizure of illegally obtained evidence from a co-conspirator. To have authority or standing to challenge the lawfulness of a search or seizure by a government agent, an individual’s personal privacy rights must be invaded (rather than those of a third party). Therefore, regardless of any violation of the co-conspirator’s rights, the defendant here has no standing to challenge the admission of the necklace. As to the defendant’s own confession, it will be inadmissible due to the violation of the defendant’s Miranda rights. The defendant clearly expressed a desire to remain silent until he could consult an attorney. Once defendant asserted this right, the re-initiation of interrogation by the police without an attorney present violated the defendant’s Fifth Amendment right to counsel. Here, the police showed defendant the diamond necklace that they had recovered and told the defendant that he had been implicated by the co-conspirator, which is conduct wherein the police knew or should have known they could get a damaging statement. Therefore, the correct answer is (B) because the defendant’s motion to exclude the necklace will be denied and defendant’s motion to exclude his own confession will be granted.
How well did you know this?
1
Not at all
2
3
4
5
Perfectly
26
Q
  1. The police received a report that women dressed as superheroes had robbed a man in his house and stolen many of his valuables. The only way onto or off the island where he lived was a ferry. The last ferry was getting ready to leave for the night when the report came in to the police.
    Without attempting to get a warrant, the police stopped all the cars waiting for the ferry. In one vehicle, the police noticed three women fidgeting rather nervously. Upon searching their auto, the police found the victim’s stolen property hidden under the front seat. The defendants were immediately placed under arrest.
    At their trial for armed robbery, the defendants move to suppress the use of the property confiscated by the police as evidence. Their motion should be
    (A) granted, because the police did not have a warrant to search their car.
    (B) granted, because the police did not have probable cause to suspect that their car contained evidence of the crime.
    (C) denied, because the police had probable cause to search their car and, under the circumstances, no warrant was required.
    (D) denied, because even though the detention was unlawful, the police had reasonable suspicion to believe that their car contained evidence of the crime since they observed the women fidgeting nervously.
A
  1. (B) Keep in mind that the warrant requirement is central to the Fourth Amendment protection against unreasonable searches and seizures. As a basic rule, all warrantLess searches are unconstitutional unless they fall into one of the following seven exceptions to the warrant requirement: (1) search incident to a Lawful arrest; (2) the “automobile” exception; (3) plain view; (4) “stop and frisk”; (5) consent; (6) hot pursuit; and (7) other emergencies. A valid warrantless search must meet the requirements of at least one of the above exceptions. With respect to the “automobile” exception, the police must have probable cause to suspect or reasonably believe that the vehicle contains evidence of the crime. In the present example, the police did not have a reasonable or articulable suspicion to believe that the defendants’ car contained evidence of the crime. As a result, choice (B) is correct because the police did not have probable cause to conduct a warrantless search of the auto. Choice (A) is wrong because, if the police had probable cause to suspect that the vehicle contained evidence of the crime, then no warrant would have been necessary. Choice (D) is incorrect because the mere fact that the police observed the women “fidgeting nervously” would not by itself give the officers probable cause to suspect that the defendants’ car contained evidence of the crime. Choice (C) is incorrect because the police did not have probable cause to search the vehicle, for reasons stated above.
How well did you know this?
1
Not at all
2
3
4
5
Perfectly
27
Q
  1. A defendant was smoking marijuana as he was driving home from work one afternoon. A police officer approached him on the freeway and stopped the car to issue a citation for speeding. As the defendant handed the officer his driver’s license, the officer smelled burnt marijuana, saw the joint, and saw an open envelope on the seat next to the defendant containing a substance that looked like marijuana. The officer ordered the defendant out of the car, arrested him for unlawful possession of marijuana. The officer then searched the defendant and found a sealed envelope in the defendant’s back pocket. Without asking the defendant’s permission, the officer opened the envelope and discovered that it contained cocaine. The officer then unzipped one of the jacket pockets on the jacket the defendant was wearing and found more cocaine and arrested the defendant for possession of cocaine.
    The defendant was taken to the police station and immediately escorted to an interrogation room. He was given his Miranda warnings, waived his rights, and gave a statement admitting to possession of cocaine and marijuana.
    Which of the following is a correct statement of the applicable legal principles regarding the search of the defendant?
    (A) When a police officer has made a lawful custodial arrest of an individual, he may, as a contemporaneous incident of that arrest, search the person of the individual.
    (B) The exclusionary rule requires that if an officer conducts an unconstitutional search, the evidence acquired in the course of the officer’s subsequent activities is inadmissible.
    (C) If an individual effectively consents to a search of their person, the evidence gathered during the search is admissible.
    (D) One who drives an automobile on a public highway does not have a legitimate expectation of privacy.
A
  1. (A) When a police officer has made a lawful custodial arrest of an individual, the police officer may, as a contemporaneous incident of that arrest, search the person of that individual. Thus, the search of the defendant’s person, after the police officer placed him under arrest, would not be violative of the Fourth or Fourteenth Amendments. The search was lawful as a result of the police officer’s probable cause to search the defendant as incident to the arrest on the charge of unlawful possession of marijuana. Choice (B) is incorrect because, as stated above, the evidence is admissible. Choice (C) is incorrect because consent is not necessary in a search incident to a lawful arrest. Choice (D) is incorrect because it is a misstatement of the law. People do still have an expectation of privacy in their persons on public roads.
How well did you know this?
1
Not at all
2
3
4
5
Perfectly
28
Q
  1. A defendant was arrested for the armed robbery of a liquor store. The defendant was taken directly from the scene of the crime in a patrol car to the police station where he was placed, still handcuffed, in an interview room. He waited for about an hour, and a detective finally came in. The detective gave the defendant his Miranda warnings, and the defendant agreed to sign the waiver and speak to the detective. The defendant then confessed to the robbery.
    Which of the following allegations would be LEAST helpful in suppressing his statement?
    (A) Before the defendant gave the statement he was refused permission to use the bathroom.
    (B) The defendant had a private lawyer on retainer at the time he gave his statement.
    (C) The defendant’s arrest was not based on probable cause.
    (D) The defendant could not speak English, and the warnings were given in English.
A
  1. (B) Choice (B) would be LEAST helpful to the defendant in suppressing his statements to the police in which he admitted robbing the liquor store. The facts tell us that the defendant waived his rights to remain silent; in accordance with Miranda, a suspect may waive his rights, as long as the waiver was knowingly, voluntarily, and intelligently made by the suspect. Consequently, the mere fact that one has retained an attorney does not preclude him from waiving his Miranda rights. Choice (A) is incorrect because it is helpful to the defendant if he is refused a chance to use the restroom. It could be seen as coercive conduct by the police. Choice (C) is incorrect because it is very helpful for the defendant if his arrest was illegal. Any confessions that proceed from it would, therefore, be inadmissible. Choice (D) is incorrect because the defendant cannot give a voluntary waiver of Miranda if he does not understand the warnings.
How well did you know this?
1
Not at all
2
3
4
5
Perfectly
29
Q
  1. A defendant was arrested and prosecuted for the crime of false pretenses. At trial, the defendant testified that she prophesied the victim’s imminent death. The defendant admitted that she told the victim she could save him if he brought a large sum of money wrapped in a handkerchief to a ritual. After the ritual, the defendant returned the handkerchief to the victim filled with waste paper rather than the money. After the defendant was convicted of false pretenses, she moved for a new trial and offered the affidavits of several jurors who sat on the case.
    An affidavit containing which of the following facts will most likely furnish the defendant’s best basis for securing a new trial?
    (A) A juror misunderstood the judge’s instructions regarding the burden of proof.
    (B) A juror, in violation of the court’s instructions, read a newspaper article implicating the defendant in several other similar schemes.
    (C) Ajuror fell asleep during defense counsel’s closing argument.
    (D) A juror admittedly stated during deliberations that he had a personal animosity toward the defendant.
A
  1. (B) The right to a fair and impartial trial is required by the concept of due process and the Sixth Amendment’s guarantee of an impartial jury. Based upon the decision in Sheppard v. Maxwell, 384 U.S. 333 (1966), choice (B) would provide the best grounds to justify a showing that a juror was biased and not impartial. In Sheppard, the U.S. Supreme Court held that the judge should act “where there is a reasonable likelihood the prejudicial news” would prevent a fair trial. Choices (A), (C), and (D) are incorrect because they have all been held not to warrant a new trial.
30
Q
  1. While on routine patrol, police officers observed a driver make an illegal U-turn. After stopping the driver’s car, they observed him reach under the driver’s seat. They ordered him out of the car and saw a cellophane package protruding from under the seat. The package contained a white powdery substance, which the officers suspected to be cocaine.
    The officers placed the driver under arrest and put him in the rear of their patrol car. They then proceeded to search the rest of the driver’s car. In the trunk they found an assault rifle that was later determined to be the weapon used in a liquor store robbery. Charged with that robbery, the driver moved to suppress the assault rifle as evidence on the grounds that the police did not have a warrant to search the trunk.
    The best theory that the prosecution can use in support of the admissibility of the assault rifle as evidence is that
    (A) the police conducted an automobile search.
    (B) the police conducted an inventory search.
    (C) the search was incident to a lawful arrest.
    (D) the search was made under exigent circumstances.
A
  1. (A) This Criminal Procedure question deals with the highly tested area of warrantless searches. By process of elimination, choice (A) furnishes the best grounds to justify the search of the driver’s vehicle. A warrantless search and seizure of items from an automobile may be permitted where there is probable cause to believe the vehicle contains contraband (or where the vehicle could be moved before there is time to obtain a warrant). In accordance with the holding in United States v. Ross, 456 U.S. 798 (1982), once probable cause to search exists, the police can search the entire vehicle, including closed containers. Choice (B) is not a strong answer, since an inventory search is one that is made either at the police station or at a police impounding. In either case, such a warrantless search is made well after the initial stopping of the vehicle. By the same token, choice (C) presents a weak argum ent because a search incident too lawful arrest extends only to the area within the immediate control of the defendant. Also, choice (D) is not the best answer because “exigent circumstances” apply only in emergency situations where the evidence may
    be lost or destroyed before a warrant can be obtained.
31
Q
  1. Sally sold heroin to John. John was later stopped by police for speeding. The police searched John’s car and found the heroin concealed under the rear seat. Sally is charged with illegally selling heroin.
    Sally’s motion to prevent introduction of the heroin into evidence will most probably be
    (A) granted, because the heroin was not in plain view.
    (B) granted, because the scope of the search was excessive.
    (C) denied, because Sally has no standing to object to the search.
    (D) denied, because the search was proper as incident to a valid full custodial arrest.
A
  1. (C) A commonly tested area on the Multistate Exam is that of standing, both in Constitutional Law, as well as in Criminal Procedure. In Rakas v. Illinois, 439 U.S. 128 (1978), a passenger who had no property interest in an automobile was held not to have standing to challenge a search of the vehicle as to items seized from it. This decision is founded on the principle of no reasonable expectation of privacy. To have Fourth Amendment standing, a person must show that his own rights were violated. Standing is proper if a person owns or has a right to possession of the place or thing searched, or if the place searched is the person’s home. Since John, not Sally, owned the car, Sally has no standing to object to the search. Choice (C) is, therefore, correct. Note further that Rakas held that being “legitimately on the premises” is insufficient grounds to assert standing by itself, without proof of some possessory interest. Also, testimony given by the defendant to assert standing may not be admitted substantively against him at trial, See Simmons v. United States, 390 U.S. 377 (1968). Choices (A), (B), and (D) are incorrect for the reasons stated above.
32
Q
  1. A reliable police informant telephoned the police to report seeing two men in a blue car selling narcotics outside a local movie theater. The informant gave a detailed description of the two men and the license number of the vehicle. A patrol car was immediately dispatched to the movie theater. A few minutes later, the police saw a car matching the description given parked about two blocks from the movie theater. When the police approached, they saw one man fitting the description provided.
    The police proceeded to the car and opened the doors of the vehicle. The police then pried open a locked tool case that was situated in the rear of the vehicle. Inside, the police found an envelope that contained about one gram of cocaine. They then placed the driver of the vehicle under arrest. Immediately thereafter, the police opened the glove compartment and found a small amount of marijuana.
    The driver of the van was subsequently prosecuted and charged with two counts of possession of controlled dangerous substances. He filed a motion to exclude the cocaine and marijuana from evidence.
    His motion will most likely be
    (A) granted, as to the cocaine only.
    (B) granted, as to the marijuana only.
    (C) granted, as to both the cocaine and the marijuana.
    (D) denied, as to the cocaine and the marijuana.
A
  1. (D) In California v. Acevedo, 59 LW 4559 (1991), the Supreme Court held that police may search an automobile and the containers within where they have probable cause to believe contraband or evidence is contained. Accordingly, in United States v. Ross, 456 U.S. 798 (1982), the Court held that if “probable cause justifies the search of a lawfully stopped vehicle, it justifies the search of every part of the vehicle and its contents that may conceal the object of the search.” Since the police had probable cause to search the entire vehicle for drugs, the police may lawfully search both the tool case and the glove compartment. Choices (A), (B), and (C) are incorrect because, for the reasons stated above, the defendant would not be successful in his motion to suppress.
33
Q
  1. Police officers received an anonymous tip that two men were selling cocaine from the back of a restaurant. The tipster stated to the police that the two men were both tall white men who were wearing jeans and black shirts, the uniform of the restaurant. The tipster also said that one of the men had red hair and was wearing a tan ball cap and the other man had black hair and was wearing a red ball cap.
    The police set up surveillance on the back of the restaurant and saw the two men make several hand- to-hand transactions. Upon receiving authority to make the arrest from their commanding officer, they waited for the two men to come out again. About two hours later, the two men came out again, this time with a woman. The officers arrested the two men and detained the woman to determine her involvement.
    The officers took the woman’s purse without her consent and searched it for illegal narcotics. In her purse they found three grams of crack cocaine, digital scales, a small glass pipe used for inhaling crack cocaine, and a balloon of black tar heroin. The woman was then arrested on the various drug charges.
    The woman was thereafter prosecuted and charged with illegal possession of a controlled dangerous substance and possession of drug paraphernalia. She filed a motion to exclude the drugs and the paraphernalia from evidence.
    Her motion will most likely be
    (A) granted, because the police did not have probable cause to conduct the search of her purse.
    (B) granted, because the police did not secure a warrant before conducting the search.
    (C) denied, because the search was justified, since she was about to leave the scene of the crime.
    (D) denied, because she was sufficiently close or proximate to the crime scene to justifi the warrantless search.
A
  1. (A) This is a tricky Multistate question that requires a careful reading of the facts. The police informant reported seeing two men engaging in the sale of narcotics. Consequently, they did not have probable cause to suspect that the woman was engaged in the commission of a crime. In this regard, it is important to point out that “a person’s expectations of privacy in personal luggage are substantially greater than in an automobile.” The rule for automobiles does not extend to personal belongings. When the circumstances are sufficiently “exigent” to allow the police to make a warrantless seizure of personal items, this does not permit the “far greater intrusion” of examining the contents thereof but only the continued possession of the personal effects while a warrant is sought. LaFave, Criminal Procedure, pg. 152. Choice (B) is incorrect because there is no probable cause to get a warrant and, therefore, Choice (A) is the better of the two choices. Choice (C) is incorrect because there are no facts to suggest that the defendant was fleeing the scene. Choice (D) is incorrect because proximity to a crime does not automatically mean probable cause exists to believe that one is an actor in the crime absent some other evidence.
34
Q
  1. A police department filed a complaint charging Bill with the sale of narcotic drugs in violation of a state statute and obtained a valid warrant for his arrest. Two police officers went to Bill’s house, a two-story colonial. After knocking at the door, they were admitted by Bill’s brother, Mark. One of the policemen showed Mark the arrest warrant and asked if he was Bill. He replied that he was Bill’s brother and that he was staying at his brother’s house while Bill was away on a business trip. The police officer then asked Mark for some identification. Mark was only able to produce an out-of-state driver’s license. The other policeman then became suspicious and proceeded to frisk Mark. During the pat-down, the police officer found the following letter, which read:
    “Dear Mark,
    If you could get rid of your brother permanently, we could be together forever.
    With love,
    your favorite girl.”
    Nervously, Mark then agreed to allow the policemen to search the house. The officers conducted an extensive two-hour search of the house and found nothing of interest to them.
    As they were leaving, one of the police officers walked over to a parked car in the driveway of the house and noticed that the car’s trunk was slightly open. When he looked in the trunk, the police officer discovered Bill’s body.
    If Mark’s attorney files a motion to suppress the introduction of the letter into evidence, the court should rule that the evidence is
    (A) admissible, because it was on the person, or in the presence of the person, who was later arrested.
    (B) admissible, because it was obtained pursuant to a valid search.
    (C) inadmissible, because the letter was obtained as a result of a non-consensual search and seizure.
    (D) inadmissible, because the letter was the fruit of an illegal search and seizure.
A
  1. (D) Choice (D) is correct, since the letter was the “fruit” of an illegal search. The Fourth Amendment prohibition against unreasonable search and seizure, as applicable to the states by the Fourteenth Amendment, would be invoked, since the officers had neither “probable cause” to search Mark nor a valid search warrant. Although “probable cause” is not required to justify an investigatory field stop (e.g., “stop and frisk”), the police still must have some objective basis for believing that the person was about to engage, or already had engaged, in criminal activity. Choices (A) and (B) are incorrect as misstatements of law. Choice (C), though correct, is the less preferred choice, since the basis for excluding the letter from evidence is that it is the “fruit” or product of an illegal search, rather than the result of a nonconsensual search.
35
Q
  1. A defendant was arrested and charged with burglarizing a pharmacy. The break-in allegedly occurred late one evening after the store had closed for business. The defendant was identified as the perpetrator of the crime by a film that was recorded during the burglary from a hidden camera. When the defendant was apprehended, he denied involvement in the crime and told the arresting officers that he had been out of town when the burglary occurred.
    Prior to trial, the defendant’s court appointed attorney filed a motion requesting discovery of the videotape film that was recorded during the perpetration of the crime. The trial judge granted the request and ordered the prosecution to send a duplicate copy of the videotape to the defendant’s attorney. Following the judge’s ruling, the prosecuting attorney, pursuant to state law, then filed a discovery motion specifically asking the defendant whether he planned to raise an alibi defense. The prosecuting attorney also sought discovery of the identity of such an alibi witness.
    Assume that the judge requires the defendant to respond as to whether the defendant intends to raise an alibi defense.
    May the judge also require the defendant to disclose the identity of the alibi witness?
    (A) Yes, because the prosecution should have an opportunity to investigate the merits of the alibi.
    (B) Yes, because the defendant waived any claim of privilege when he sought discovery of the film from the prosecution.
    (C) No, because by requiring the defendant to reveal information before he is tactically ready to do so substantially impairs his ability to successfully defend himself.
    (D) No, because such disclosure constitutes an implied representation that is testimonial in character and, thus, violates the defendant’s privilege against self-incrimination.
A
  1. (A) Although state courts are not required to permit liberal pretrial discovery, they must grant the defense the same discovery rights given to the prosecution. This evenhandedness is also apparent with regard to the disclosure of the identity of alibi witnesses where the defendant intends to raise such a defense. Many states require, by statute or rule of court, that a defendant who intends to rely on alibi as a defense should give the prosecution advance notice of his intention, and often includes the disclosure of the identity of the alibi. The purpose of these statutes is to afford the prosecution an opportunity to investigate the merits of the alibi, a defense which is easily fabricated and manufactured. Holding that these statutes are valid, courts have rejected claims that the enactments are unconstitutional in that requiring the defendant to make an advance disclosure of his alibi defense denies him due process of law, equal protection of the laws, or violates his privilege against self-incrimination. Here, because the facts indicate that there is a state law basis for the request by the prosecution, answer choice (A) is correct, as it comes to the correct conclusion and relies on the essential reason for these enactments. Choices (B), (C), and (D) are incorrect for the reasons stated above.
36
Q
  1. A woman was driving her van along a public road one night. A police officer, who was driving behind the woman, decided to make a random stop of the woman’s vehicle to check her license and registration. The officer pulled the woman’s van over to the side of the road and then walked up to the driver’s side of the vehicle. When he came alongside the driver’s window, the officer asked the woman for her identification. As the woman was thumbing through her wallet, the officer shone his flashlight into the van and spotted a plastic bag containing marijuana lying on the floor under the back seat. The officer then arrested the woman and charged her with possession of marijuana.
    At the woman’s trial for illegal possession of a controlled substance, her attorney moved to suppress the use of the marijuana as evidence. Her motion should be
    (A) granted, because the marijuana was the fruit of an illegal search.
    (B) granted, because the police officer did not have probable cause or a reasonable suspicion to believe that the woman’s van contained a controlled substance.
    (C) denied, because the marijuana was in plain view when the police officer shone his flashlight inside the van.
    (D) denied, because the seizure of the marijuana was made pursuant to a lawful investigatory stop.
A
  1. (A) A random stopping of a vehicle on the highway where the officer has no suspicion of wrongdoing is unconstitutional because it leaves too much discretion to the police officer. See Delaware v. Prouse, 440 U.S. 648 (1979). And, applying the “fruits of the poisonous tree” doctrine, no evidence seized as a result of a Fourth Amendment violation may be admitted at trial. See Wong Sun v. United States, 371 U.S. 471 (1963). Note: The rule suppressing fruits of an illegal search applies not only to objects found, but also to verbal statements obtained because of the original tainted search or as a result of an illegal arrest. See Brown v. Illinois, 422 U.S. 590 (1975). Choices (B), (C), and (D) are incorrect for the reasons stated above.
37
Q
  1. A state has recently enacted a statute making it a misdemeanor for any person to smoke a cigarette, cigar, or pipe in any restaurant, bar, cafe, or other establishment within the state.
    A defendant was tried in state court for violating the anti-smoking statute. The prosecution’s evidence consisted of testimony regarding the defendant’s cigarette smoking at a restaurant. During the prosecution’s case-in-chief, they called a witness who testified to a prior conviction of the defendant that had been excluded by the judge in a pretrial hearing. The judge immediately granted a mistrial and excused the jury.
    The state prosecutor appealed the ruling under a statute that permitted prosecutorial appeals in such circumstances. The state won the appeal, but the prosecutor decided not to re-prosecute the defendant. However, the city attorney from the city in which the restaurant was located then sought to prosecute the defendant for the same incident under the city’s anti-smoking ordinance, which was identical to the state statute. The city attorney planned to call as her only witness a man who testified at the defendant’s first trial. The defendant moved to dismiss on the grounds that the prosecution of the city charge would violate his rights against double jeopardy.
    The court should
    (A) grant the motion, because jeopardy attached in the first prosecution.
    (B) grant the motion, because the law and the evidence will be identical at the second trial.
    (C) deny the motion, because the city and the state are separate sovereigns.
    (D) deny the motion, because the judge granted a mistrial.
A
  1. (D) The Fifth Amendment right to be free from double jeopardy for the same offense prohibits retrial after a determination on the merits. In United States v. Scott, 437 U.S. 82 (1978), the Court held that “the defendant, by deliberateLy choosing to seek termination of the proceedings against him on a basis unrelated to factual guilt or innocence of the offense of which he is accused, suffers no injury cognizable under the Double Jeopardy Clause if the Government is permitted to appear from such a ruling ofthe trial court in favor of the defendant.” LaFave and Israel, Criminal Procedure, pg. 910. Thus, the general rule is that reprosecution following dismissal bythe judge upon the defendant’s motion that does not constitute an acquittal on the merits is not prohibited bythe double jeopardyclause. In these facts, the defendant’s motion for dismissal at the first trial was granted by the judge based on a witness testifying to previously excluded evidence, not on any factual element of the statutory offense charged. Hence, no jeopardy has attached to barthe defendant’s municipal prosecution. Choice (A) is incorrect, and the defendant’s motion to dismiss atthe second trial will be denied. Choice (D) properly states the correct rationale. Choice (B) is incorrect because collateral estoppel bars retrial of issues that have been actually litigated. The defendant’s first trial ended in a dismissal without litigating any factual issues not being raised at the second trial. Choice (C) is incorrect because the separate sovereignties doctrine does not apply to trials by a state and by its municipalities.
38
Q
  1. While parked on a downtown street, a police officer noticed a man sauntering down the street, whistling. As the man came under a street light, the officer recognized him as a parolee from a neighboring state. The officer got out, stood in front of the man, and asked him to give an account of himself. The man replied, “I’m the president, you dope … get out of my way or the Secret Service will gun you down,” and reached into his coat pocket to bring something out, which he held in his clenched fist. The officer forced the man’s hand open and found a number of diamond rings therein. Just then, the police radio in the officer’s car announced that the burglar alarm of a jewelry store had rung in police headquarters. Believing that the man was responsible for burglarizing the jewelry store, the officer arrested him and took him to the police station where he was booked and fingerprinted. After he was given his Miranda warnings, the man requested to speak to his attorney. The officer led the man to a telephone and asked him if he knew his attorney’s phone number. The man responded and gave the officer a phone number. The officer dialed the number and waited until a voice answered, “law offices.” The officer then handed the telephone receiver to the man. As the officer was walking out of the room, he heard the man say, “Hello, it’s me. I just got arrested after robbing a jewelry store.”
    At trial, the officer is called to testify to what the man told his attorney during their telephone conversation. Upon proper objection by the man’s attorney, the officer’s proposed testimony should be
    (A) admitted, because the man’s confession was not coerced.
    (B) admitted, because the statement was not the product of interrogation.
    (C) excluded, because the officer’s conduct violated the man’s Sixth Amendment right to counsel.
    (D) excluded, because the officer’s conduct violated the man’s attorney-client privilege.
A
  1. (B) There are four specific bases with which to attack the admissibility ofa statement or confession: (1) the Miranda standard; (2) the right to counsel approach; (3) the voluntariness approach; and (4) fruits of illegal conduct. The Miranda standard applies only once custodial interrogation has begun. Statements elicited once the right to remain silent has been requested or once counsel has been requested will be inadmissible. In this question, the man was arrested and given his Miranda warnings. He requested counsel. However, his statements to his attorney were not made as a product of interrogation. The man’s statement was not violative of the Miranda standard once one assumes the risk that a telephone conversation will be overheard. Therefore, the officer’s testimony will be admitted under choice (B). Choice (A) is incorrect because the man’s remarks were inadvertent; he was not knowingly and voluntarily conferring, so lack of coercion is not the proper basis upon which to admit his statement. Choice (C) is incorrect. The Sixth Amendment right to counsel approach, which attaches to all “critical stages” of a criminal proceeding, operates to exclude any statements deliberately elicited once criminal charges have been filed. The man’s statements were volunteered, not elicited. Finally, choice (D) is incorrect because the man, as holder of the attorney-client privilege, waived confidentiality by speaking out over the telephone. Note: General on-the-scene questioning as to facts surrounding a crime or other general questioning of citizens in the fact-finding process is not considered custodial interrogation for Miranda purposes. See LaFave, Criminal Procedure, pg. 292.
39
Q
  1. A defendant was booked on a commercial airline flight. When the defendant arrived at the airport, two undercover police narcotics agents observed him. His appearance and actions led the police to believe that he fit the description of a predetermined profile of a drug courier. The officers approached the defendant and told him they wanted to detain him for questioning. The defendant asked what for, and the police explained that they were narcotics agents assigned to the airport and that he fit the profile of a drug courier. They then asked the defendant to hand over his overnight bag. The defendant refused. The officers then explained that he wouldn’t be permitted to board the plane unless they inspected the contents of his bag. The defendant told the officers that he changed his mind and decided not to fly today after all. The officers still requested that the defendant turn his overnight bag over to them. The defendant did so, and then he was given permission to leave the airport.
    The next day, the police had a dog sniff the defendant’s bag for narcotics. The results of this search gave police suspicion to believe that the bag contained narcotics. They opened the defendant’s bag and found heroin inside. The defendant was subsequently arrested and charged with unlawful possession of narcotics.
    At trial, the defendant’s attorney moves to prevent introduction of the heroin into evidence. This motion will most likely be
    (A) granted, because the police did not have probable cause to hold the bag overnight for the search.
    (B) granted, because the heroin was discovered as a result of an unlawful airport detention.
    (C) denied, because the defendant fit the predetermined profile of a narcotics courier.
    (D) denied, because the search resulted from a lawful airport detention.
A
  1. (A) Here’s a tricky Criminal Procedure example dealing with airport searches. Generally, under federal law, warrantless administrative searches are permitted at airports in order to protect passengers from weapons and explosives. Although narcotics- sniffing dogs may sniff a passenger’s luggage, any resulting detention must be brief and a seizure of the luggage is subject to Fourth Amendment limitations. See United States v. Place, 77 L.Ed. 2d 110 (1983). Although the police may have been justified to temporarily detain the defendant, they would not be permitted to confiscate his baggage and hold it overnight. Choice (A) is correct because such a seizure is violative of the Fourth Amendment based on the decision in the Place case. Choice (B) is incorrect because the initial detention of the defendant was lawful. Choices (C) and (D) are incorrect for the reasons stated above.
40
Q
  1. A man was standing on a street corner when a police officer approached him. The officer gently grabbed the man’s arm and said, “Hey, I want to talk to you.” The man responded, “About what … I’m in a hurry to go home and watch the baseball game.” The officer then remarked, “Listen, what were you doing at the liquor store last week?” The man stated, “All right, man, I was there during the robbery.” Whereupon, the officer placed the man under arrest and charged him with robbery.
    The man was taken to police headquarters where he was booked. For the first time, he was given his Miranda warnings. He then waived his rights and confessed to being an accessory during the liquor store robbery. Afterward, the court appointed an attorney to represent the man. The attorney moved to suppress both statements the man gave to the police. The trial court granted the motion with respect to the first statement as a violation of Miranda. The man’s attorney now moves to suppress the second
    statement given at the police headquarters.
    This motion should be
    (A) granted, because the second statement was the fruit of the first statement.
    (B) granted, because the first statement was excluded, and the second statement is tainted.
    (C) not granted, because the man had already confessed to the crime.
    (D) not granted, because the second statement was volunteered after a knowing Miranda waiver.
A
  1. (A) In Missouri v. Seibert, 542 U.S. 600 (2004), the Supreme Court ruled (5-4) that a police officer’s intentional, bad-faith decision to interrogate a suspect in custody without first giving the required Miranda warnings, which produced a confession followed by Miranda warnings, renders a second post-Miranda confession also inadmissible. Justice Kennedy, who provided the fifth vote based his decision on the issue of whether the police deliberately withheld Miranda warnings initially in order to obtain a second confession (which followed valid warnings close in time to the first unwarned confession). The man’s first unwarned confession is inadmissible as a direct violation of Miranda. The man’s subsequent post-Miranda confession must also be suppressed because the officer deliberately withheld Miranda warnings prior to his first confession. Therefore, the man’s subsequent confession must also be suppressed. Choices (B), (C), and CD) are incorrect for the reasons stated above.
41
Q
  1. Two men held-up a liquor store in a city. During the robbery, one of the participants shot and killed the owner of the store. Over a year later, police detectives arrested a defendant after obtaining reliable information that the defendant was the robber who was responsible for killing the store’s owner. Afterward, the defendant was taken to the station house where he was booked for the crime and then incarcerated. The next day, the prosecuting attorney made a decision to delay indictment until he could discover the identity of the defendant’s accomplice. The defendant was then released from police custody. In this jurisdiction the statute of limitations for murder is five years.
    Five months later, the prosecuting attorney, after unsuccessfully attempting to secure information on the defendant’s cohort, indicted the defendant, charging him with felony murder. Shortly before trial, the only eyewitness to the crime died. He would have testified that the defendant did not participate in the robbery.
    The defendant’s motion to dismiss the indictment because of the delay between the date of the crime and the date of the indictment will most probably be
    (A) granted, because the prosecutor is constitutionally required to press charges, if at all, within a reasonable time after probable cause is established.
    (B) granted, because the delay in prosecuting the defendant actually prejudiced him.
    (C) denied, because pre-indictment delay does not violate the Constitution, as long as it does not exceed an applicable statute of limitations.
    (D) denied, because the delay was not excessive and was for a permissible purpose.
A
  1. (D) Another area in Criminal Procedure commonly tested on the bar exam deals with indictment delays after arrest. When a defendant, after being arrested and released, is not indicted until a long time interval has passed, the Sixth Amendment right to a speedy trial does apply. See Dillingham v. United States, 423 U.S. 64 (1975). However, the Court determines whether the delay is reasonable or unreasonable by balancing the following factors: (1) the length of the delay; (2) the good faith and justification of the delay; (3) the defendant’s assertion of the right to a speedy trial; and (4) prejudice to the defendant. The prosecution’s delay in indicting the defendant was for a proper purpose (i.e., to obtain information about his accomplice). Moreover, a five-month delay (where the statute of limitations is five years for murder) does not appear to be unreasonably long. Therefore, choice (D) is correct. Choices (A), (B), and (C) are incorrect for the reasons stated above.
42
Q
  1. A state has adopted a system of bifurcated trials in cases in which a defendant’s insanity is in issue. According to the bifurcated trial system, whenever a defendant pleads not guilty to an offense by reason of insanity, two trials will be held. The first one will simply determine whether the defendant has committed the offense for which she is charged. This trial will not address the issue of insanity. In the event that it is found that the defendant has, in fact, committed the offense, then a second trial will be conducted to determine whether she should be exculpated for the criminal action by reason of insanity.
    A woman was arrested and charged with murder. She pleaded not guilty by reason of insanity. At her first trial, the state introduced evidence showing that the woman was having an affair with the victim. When the victim tried to break off their relationship, the woman shot and killed him during a lover’s quarrel. The woman was then called to testify in her own behalf. She testified that she had been living with the victim for two years prior to the time of his death. During that period she had undergone psychiatric treatment and was diagnosed as being schizophrenic. She further testified that at the time the victim was killed, she was under the influence of narcotics. While she was hallucinating, she remembered perceiving the victim as a demon and shot at this satanic figure in order to free herself from his evil spell. She then testified that she didn’t believe shooting the demon was morally wrong. The prosecuting attorney objected to the woman’s testimony. Over such objections, the trial judge admitted the woman’s testimony.
    Was the trial judge correct in admitting the woman’s testimony?
    (A) No, because proof of mental disease requires the use of expert testimony.
    (B) No, because testimony relating to her belief that she didn’t know what she was doing was wrong, is not relevant until the second trial.
    (C) Yes, because her testimony is relevant to the mental state necessary for the commission of the crime.
    (D) Yes, because her testimony is relevant to the issue of self-defense.
A
  1. (C) Students must be familiar with the area of “bifurcated trials.” In a very few jurisdictions, most notably California, the defense of insanity is tried separately from the other issues in the case. Under California law, for example, a defendant may plead (1) not guilty, (2) not guilty and also not guilty by reason of insanity, or (3) merely not guilty by reason of insanity. If the second form of plea is entered, the guilty stage of the trial is first concluded without any reference to the insanity defense, after which (if defendant was found guilty) a separate proceeding takes place before the same or a different jury for purposes of trying the insanity defense. According to LaFave, pp. 315-316, the purpose of the bifurcated trial procedure is to eliminate from the basic trial on the issue of whether the defendant engaged in the conduct a great mass of evidence having no bearing on that question and which may confuse the jury or be made the basis of appeals to the sympathy or prejudice of the jury. However, this objective has not been realized. Evidence of mental disease or defect is admissible on the issue of whether the defendant had the requisite mental state and is permitted at both the guilty stage and insanity stage of the trial. Choices (A), (B), and (D) are incorrect for the reasons stated above.
43
Q
  1. A defendant is arrested and charged with shoplifting. He claims that he intended to pay for the item he took and requests an attorney and a jury trial. Both requests are denied by the judge, and the defendant is found guilty. The maximum penalty for shoplifting is a $500 fine and six months in jail. The defendant is sentenced to three months in jail, which he is allowed to serve on weekends in the county detention center.
    The defendant claims that his constitutional rights have been violated. Is he correct?
    (A) No, because his actual sentence was only three months.
    (B) Yes, because the defendant was denied the right to counsel.
    (C) Yes, because the defendant was denied the right to a jury trial.
    (D) Yes, because the defendant was denied both the right to counsel and the right to a jury trial.
A
  1. (B) The Sixth Amendment provides that the accused shall have “the assistance of counsel for his defense.” This right not only encompasses the right to hire private counsel, but also the right to be provided with counsel without charge if the accused is unable to afford counsel. Choice (A) is incorrect because in Argersinger v. Hamlin, 407 U.S. 25 (1972), the U.S. Supreme Court held that “absent a knowing and intelligent waiver, no person may be imprisoned for any offense, whether classified as petty, misdemeanor, or felony unless he was represented by counsel.” Note that choices (C) and (D) are not correct because the right to jury exists whenever the accused faces a sentence of possibly more than six months. See Baldwin v. New York, 399 U.S. 66 (1970). In this example, since the defendant’s maximum sentence was not more than six months in jail, he would not have the right to a jury trial.
44
Q
  1. A defendant was a senior pre-med student at a state university. Before a big football game with their arch rival, the defendant attended a pep rally at the university football field. During the pep rally, the students drank beer, sang songs, and gave members of the football team fervent cheers. After the rally ended, the defendant was walking toward his car when he felt the need to relieve himself. He then strutted across the street to a municipal park and urinated against a tree. In this jurisdiction, urinating in public is a misdemeanor.
    Unknown to the defendant, a parking enforcement officer who was ticketing cars in the area witnessed the incident and wrote down the license number of the defendant’s vehicle as he drove off. She immediately gave the information to the police who did a check and learned the defendant’s identity and home address. Within an hour, the police went to the defendant’s house and forcibly entered the dwelling without a search warrant. While searching his home, they found the defendant studying in an upstairs bedroom. After giving him his Miranda warnings, the defendant confessed to the crime.
    The defendant was thereafter prosecuted for the misdemeanor of urinating in public. If the defendant moves to suppress evidence of the statement that he made to the police when he was apprehended, the motion should be
    (A) denied, because the defendant received Miranda warnings and was not compelled to incriminate himself.
    (B) denied, because the defendant volunteered the confession.
    (C) sustained, because the police lacked probable cause to search the defendant’s home.
    (D) sustained, because the defendant’s statement was the product of a warrantless entry of his home.
A
  1. (D) The exclusionary rule is a remedy for Fourth, Fifth, and Sixth Amendment violations by which all evidence obtained through illegal searches and seizures is inadmissible. The scope of the rule applies furthermore to exclude all evidence obtained orderived therefrom. Such additional evidence is deemed to be tainted “fruit of the poisonous tree.” See Wong Sun v. U.S., 371 U.S. 471 (1963). The forcible entry and search by the police of the defendant’s home was illegal, since they did not obtain a warrant when the law required them to do so. As a result of their illegal, warrantless search of the home, the defendant’s confession will be properly excluded underthe fruit of the poisonous tree doctrine. Choice (D) is correct. Choice (C) is incorrect because it gives the wrong justification for excluding the defendant’s confession. Regarding choices (A) and (B), which reach the wrong conclusion, even where the Miranda warnings are given afterthe illegal police conduct but before the confession, the confession is still regarded as inadmissible fruit. See Taylor v.Alabama, 457 U.S. 687 (1982).
45
Q
  1. A defendant, who was an indigent, was arrested and charged with possession of less than one ounce of marijuana. Under the relevant state statute, possession of less than one ounce of marijuana is a misdemeanor, punishable by a maximum of one year in jail and/or a fine of $500. At trial, the defendant pleaded not guilty and requested that an attorney be appointed to represent him. The trial judge refused to honor the defendant’s request for an attorney.
    If the defendant is subsequently found guilty, which of the following is the most severe sentence that can constitutionally be imposed?
    a. A $500 fine.
    b. Six months in jail.
    c. One year in jail.
    d. No sentence, because the defendant was denied
    the right to counsel.
A
  1. (A) In Argersinger v. Hamlin, 407 U.S. 25 (1972), the U.S. Supreme Court held that “no person may be imprisoned for any offense, whether classified as petty, misdemeanor, or felony unless he was represented by counsel.” Also be aware that in Scott v. Illinois, 440 U.S. 367 (1979), the U.S. Supreme Court declined to extend Argersinger to a case where one is charged with an offense for which imprisonment upon conviction is authorized but not actually imposed. In Scott, the petitioner, an indigent, was charged with shoplifting merchandise valued at less than $150, punishable by as much as a $500 fine, or one year in jail, or both. He was not provided counsel. After a bench trial, he was convicted of the offense and fined $50. Choices (B), (C), and (D) are incorrect for the reasons stated above.
46
Q
  1. One afternoon after school, a 14-year-old girl went into a drug store. While browsing, she picked up a candy bar and placed it in her coat pocket. She left the store without paying for it. Unknown to the girl, the store security guard saw what she had done. He immediately ran outside, grabbed her arm, and accused her of stealing the candy bar. The girl broke down and started crying. She then told the security guard that she was hungry because she hadn’t eaten any lunch.
    At a juvenile delinquency court hearing, the security guard proposes to testify to what the girl said. Upon objection by the girl’s attorney, the strongest reason for admitting her statement will be because
    (A) Miranda warnings need not be given to juveniles.
    (B) the security guard’s remark was an accusation, not an interrogation.
    (C) the security guard was not employed by an agency of the government.
    (D) the girl was not in custody.
A
  1. (C) The Fourth Amendment generally protects only against governmental conduct and not against searches by private persons. Be advised that government agents include only the publicly paid police and those citizens acting at their direction or behest, and not private security guards unless deputized as officers of the public police. Also, the same requirement of governmental conduct in Fourth Amendment cases applies to confession cases as well. Since Miranda applies only to interrogation by the publicly paid police (or private citizens acting at their behest), choice (C) is the best answer. Choice (A) is incorrect because it is a misstatement of law. Choice (C) is a stronger answer than choices (B) and (D) because if Miranda is not applicable, then it is irrelevant whether or not there was a custodial interrogation.
47
Q
  1. A defendant was arrested and indicted separately for the crimes of robbery, rape, and felony murder with the underlying felony being the rape.
    With respect to double jeopardy, which of the following statements is most accurate?
    (A) The defendant may be tried for each offense separately and may be convicted of each, with sentences running consecutively.
    (B) The defendant may be tried for the two separate offenses of robbery and felony murder and may be convicted of both, with sentences running consecutively.
    (C) The defendant may be tried for the two separate offenses of robbery and felony murder and may be convicted of both, with the robbery sentence running consecutively with concurrent sentences for the felony murder and rape.
    (D) The defendant may be tried for the single offense of felony murder and sentenced for that crime only.
A
  1. (C) Although many students will undoubtedly choose choice (B) as correct, choice (C) is preferred. First, the double jeopardy rule prohibits multiple punishments for the same criminal offense. However, where a defendant is convicted of two separate criminal offenses (e.g., where each crime requires proof of a fact that the other does not), then consecutive sentences (or multiple punishments) may be imposed. That’s why the defendant may be convicted of the separate crimes of armed robbery and felony murder with sentences running consecutively. On the other hand, the defendant could not receive consecutive sentences for rape and felony murder because rape is a lesser included offense inasmuch as rape was the underlying felony. However, the court could still impose concurrent sentences for rape and felony murder because concurrent sentences are not multiple sentences. Therefore, there is no violation of the double jeopardy rule. Choices (A), (B), and (D) are incorrect for the reasons stated above.
48
Q
  1. A boyfriend and his girlfriend broke into a house late at night with the intent to steal a stereo system. Although they believed that the homeowner was away on a business trip, he was, in fact, asleep in an upstairs bedroom. While they were inside the house, the girlfriend announced that she had changed her mind and urged her boyfriend to leave.
    The homeowner, who was awakened by the noise downstairs, descended the staircase to investigate. Upon seeing the homeowner, the girlfriend again urged her boyfriend to flee. Instead, the boyfriend attacked the homeowner and tied him up with rope. Thereupon, the boyfriend and girlfriend departed with the stereo equipment. After they left, the homeowner choked to death from the ropes while trying to free himself.
    The boyfriend and girlfriend were charged with burglary but were acquitted. Thereafter, the boyfriend and girlfriend were apprehended and prosecuted for felony murder with the underlying felony being burglary.
    With respect to the boyfriend and girlfriend’s criminal liability for felony murder, which of the following is most correct?
    (A) The burglary acquittal precludes any subsequent prosecution under the doctrine of res judicata.
    (B) The burglary acquittal precludes any subsequent prosecution under the doctrine of collateral estoppel.
    (C) The burglary acquittal precludes prosecution for felony murder under the doctrine of double jeopardy.
    (D) The burglary acquittal does not preclude subsequent prosecution for felony murder.
A
  1. (C) This is a tricky double jeopardy question concerning whether a defendant may be prosecuted and punished for both felony murder and the underlying felony. One double jeopardy issue concerns when multiple prosecutions may be undertaken, as to which the Supreme Court in Brown v. Ohio, 432 U.S. 161 (1977), adopted the longstanding Blockburger test, which originated as a device for determining congressional intent as to cumulative sentencing: “The applicable rule is that where the same act or transaction constitutes a violation of two distinct statutory provisions, the test to be applied to determine whether there are two offenses or one, is whether each provision requires proof of an additional fact which the other does not.” This means, as the Supreme Court held in Harris v. Oklahoma, 433 U.S. 682 (1977), that except in extraordinary circumstances, a defendant may not constitutionally be separately tried for felonymurder and the underlying felony. Choice (A) is incorrect because res judicata applies only to civil cases. Choice (B) is incorrect because although collateraL estoppel has been an established rule of federal criminal law for more than 50 years, we are not dealing with issue preclusion (e.g., identity or motive) from a prior trial. Conversely, we are dealing with whether the burglary acquittal precludes subsequent prosecution for felony murder. Finally, choice (D) is incorrect forthe reasons stated above.
49
Q
  1. A suspected drug dealer was arrested after selling three grams of cocaine to undercover police agents. He was subsequently convicted in state court of possession with intent to distribute narcotics. Following his conviction, the drug dealer was sentenced to a prison term of 10 years. After being sentenced, the drug dealer’s attorney appealed the conviction, citing jury misconduct. While his appeal was pending, the drug dealer was then indicted by both a federal and a state grand jury for conspiracy to distribute the same cocaine.
    The drug dealer’s attorney filed motions to dismiss each of the new indictments on the grounds that they violate double jeopardy. In all likelihood, double jeopardy requires the dismissal of
    (A) both indictments.
    (B) the state indictment, but not the federal indictment.
    (C) the federal indictment, but not the state indictment.
    (D) neither of the indictments.
A
  1. (D) In sum, the double jeopardy provision of the Fifth Amendment provides that no person shall be “twice put in jeopardy” for the “same offense.” According to the prevailing view, it has been held that two crimes are not the “same offense” merely because they arose out of the same transaction. Choice (D) is correct because successive state and federal prosecutions for the same acts are constitutionally permissible, as each sovereign must be free to vindicate its own interests. Choices (A), (B), and (C) are incorrect for the reasons stated above.
50
Q
  1. The police received an anonymous tip informing them that a pharmacist was engaged in the illegal manufacture of synthetic cocaine. As part of its investigation, the police placed an electronic tracking device on the pharmacist’s car. The tracking device was attached to the underbody of the pharmacist’s car while it was parked outside his home. The police did not secure a warrant before installing the device.
    By means of the tracking device, the police were able to trail the pharmacist’s movements. The police followed the pharmacist every day for almost a month. Finally, one day the police tracked the pharmacist’s car to a vacant warehouse on the outskirts of town. While the pharmacist was inside the building, the police peered in the window and saw drug paraphernalia and equipment used in the manufacture of synthetic cocaine.
    Based on these observations, the police secured a search warrant and gained entry into the building. Once inside, the police arrested the pharmacist and confiscated a large quantity of synthetic cocaine that had just been produced.
    At his trial for illegal possession and manufacture of a controlled dangerous substance, the pharmacist moves to suppress the cocaine confiscated by the police. The pharmacist’s motion will most likely be
    (A) granted, because the information upon which the search warrant was based was illegally obtained by means of the tracking device.
    (B) granted, because the seizure must be suppressed as the fruit of an illegal search.
    (C) denied, because the police could have discovered the location of the warehouse simply by following the pharmacist’s car.
    (D) denied, because the electronic surveillance of the pharmacist’s car did not exceed 30 days.
A
  1. (C) Another search and seizure technique that has been tested on the MBE involves the use of an electronic tracking device, such as a beacon or “beeper.” Such beepers are attached to a car, airplane, or container, and the movements of that object are then tracked by the police. In United States v. Karo, 468 U.S. 705 (1984), the Supreme Court concluded that “the mere installation of a beeper is no search because that act alone infringed no privacy interest.” The Court reasoned that the surveillance objected to “amounted principally to the following of an automobile on public streets and highways, which if accomplished merely by visual surveillance would be no search because one traveling in an automobile on public thoroughfares has no reasonable expectations of privacy in his movements from one place to another.” Therefore, choices (A) and (B) are incorrect. Choice (D) is incorrect because the length of the surveillance is not a relevant factor.
51
Q
  1. A man and a woman were arrested and charged with bank robbery. After receiving Miranda warnings, both requested an attorney. Following their arrests, the man and the woman were placed in separate jail cells.
    The next day before arraignment, the man asked a police officer if he could see his friend, the woman. The man was taken to the woman’s jail cell and allowed to talk to her. Neither defendant knew that the police had placed a listening device in the woman’s jail cell. As a result, their conversation was being overheard and recorded by the police. The police planted the listening device without first securing a warrant.
    While they were together, the man told the woman not to admit to anything because they could beat the charges with the help of a good lawyer. At trial, the prosecution sought to introduce the man’s incriminating statement into evidence. The man’s attorney has filed a motion to exclude.
    The motion should be
    (A) denied, because the defendants did not have a reasonable expectation of privacy.
    (B) denied, because the statement did not violate the man’s Fifth Amendment privilege against self-incrimination.
    (C) granted, because both defendants requested an attorney before engaging in their conversation.
    (D) granted, because the police did not secure a warrant before installing the listening device.
A
  1. (A) In determining the admissibility of a confession, the Supreme Court follows the “voluntariness” test, namely, whether the confession was “voluntary” or the product of police coercion. The Fourteenth Amendment’s due process voluntariness test requires examination of the “totality of circumstances” surrounding the confession. Generally speaking, this means that it is necessary to assess the characteristics and status of the person who gave the confession and also the conduct of the police in obtaining it. In this question, the man gave his confession freely without being subjected to any coercive police practices. As a result, choice (A) is correct inasmuch as there is no privacy expectation interest in the confines of a jail cell. Choice (D) is wrong because wiretapping and other forms of electronic surveillance are unlawfu I if they violate a person’s reasonable expectation of privacy. Always remember that in order to have a Fourth Amendment right, a person must have a reasonable expectation of privacy with respect to the place searched or the item seized. There is no legitimate expectation of privacy in a jail cell, especially given the fact that the defendant made the request to speak to his co-conspirator. Choices (B) and (C) are incorrect for the reasons stated above.
52
Q
  1. Government drug officials from a neighboring country contacted a federal agency notifying the agency that drugs were being shipped by express mail next-day service to a defendant. The federal agency contacted the local police with the information regarding the defendant. The local police had long suspected the defendant of dealing drugs.
    The next day, two undercover police officers drove to the defendant’s home and waited outside. A couple of hours later, the officers saw an express mail delivery truck pull up outside the defendant’s home. The delivery person carried an express mail box to the front door and rang the doorbell. The officers then witnessed the defendant answer the door and take the package inside.
    Approximately 20 minutes later, the defendant left the house carrying the express mail box. She walked to her car, placed the package in the trunk, and drove off. The police followed in the squad car and pulled her over at the next traffic signal. The officers placed the defendant under arrest and instructed her to open the trunk. They confiscated the express mail box and opened it. Inside was a package of white powder, which lab tests later confirmed to be cocaine.
    The defendant was charged with illegal possession of cocaine. While awaiting trial, her attorney filed a motion to suppress the introduction of the cocaine into evidence. The motion will be
    (A) granted, because the police failed to obtain a search warrant before opening the trunk of the vehicle.
    (B) granted, because the police failed to obtain a search warrant before opening the package.
    (C) denied, because the police had probable cause to conduct the search.
    (D) denied, because the search was incident to a lawful arrest.
A
  1. (C) Choices (A) and (B) are incorrect. Under the automobile exception to the warrant requirement, in United States v. Ross, 456 U.S. 798 (1982), the Court ruled thatwhen the police have probable cause to justify the search of an automobile, they may search the entire car and open any packages or containers found that may contain the items for which they have probable cause to search. In California v. Acevedo, 500 U.S. 565 (1991), the Court held that the police can also search without a warrant any container that they have probable cause to search once it is placed in a car. In this question, the police had probable cause to search the express mail box placed in the vehicle by the defendant. Based upon the holding in Acevedo, the search of the box and seizure of the cocaine was, thus, lawful. Choice (C) is correct. Choice (D) is incorrect because the police had probable cause to search the vehicle regardless of any arrest that may have preceded the search.
53
Q
  1. Police were conducting a house-to-house search for an escaped convict. During the search, a police officer went to a home and rang the doorbell. When a teenage male opened the door, the police officer told him that a convict had escaped and asked if he had seen anyone suspicious in the neighborhood. The teenager said he hadn’t but then asked the police officer to come inside to ensure the house was safe.
    The police officer entered the home and proceeded to look around. When he went into the kitchen, the police officer noticed a bag containing white powder on the counter. Suspecting that the bag contained narcotics, the police officer questioned the teenager as to its contents. The teenager said he didn’t live in the house and had never seen it before. The police officer confiscated the bag and took the teenager to the station house for questioning, at which time it was determined that the teenager had escaped from a psychiatric hospital and had broken into the home earlier in the day. Furthermore, the results of a police lab test found that the bag contained heroin. As a result, the police issued an arrest warrant for the owner of the home. He was apprehended and charged with possession of heroin.
    The homeowner’s attorney has filed a motion to suppress, claiming that the heroin confiscated by the police was unlawfully obtained. The motion should be
    (A) granted, because the teenager did not have the authority to consent to a search of the house.
    (B) granted, because the teenager did not have an ownership or shared occupancy interest in the premises.
    (C) denied, because the police officer reasonably believed that he had permission to enter the dwelling.
    (D) denied, because the police officer reasonably believed that he had permission to enter the dwelling and reasonably believed that the teenager was competent to give consent.
A
  1. (A) Consent searches now constitute one of the hottest areas in Criminal Procedure on the Multistate. With respect to third-party consent cases, any person with an equal right of control (or who possessed “common authority”) over the premises generally may consent to a search. The Supreme Court has adopted two bases for its “common authority” rule: (1) that the consenting party could permit the search “in his own right”; and (2) the defendant had “assumed the risk” that a co-occupant might permit a search. Here, the teenager did not have authority to consent to the police search. He did not have the owner’s permission to be on the premises and had unlawfully broken into the home earlier in the day. Because the owner did not “assume the risk,” the consent was invalid. Choice (A) is correct. Choice (B) is incorrect because there are situations where a third party may consentto a search even though he does not have a shared occupancy interest in the premises (e.g., school official consents to search of student’s locker). Choices (C) and (D) are incorrect because there is not enough on these facts to show that the police officer has a reasonable belief that the teenager had the apparent authority to consent to the search.
54
Q
  1. A mother has an 11-year-old daughter. The mother is legally separated from her husband and has filed for divorce. The mother and her husband own their home in joint tenancy. One night, the mother went out to dinner and left her daughter at home. Upon leaving the house, the mother locked the door to her bedroom.
    While the mother was gone, the daughter found a spare key to her mother’s bedroom and opened the door. While rummaging through her mother’s dresser, she found a bag containing white powder. Concerned that the bag might contain drugs, the daughter called her father to ask his advice. The husband instructed her to call the police. The daughter then telephoned the police station and reported what she had found. A patrol car was immediately dispatched to her home.
    The daughter allowed the police to come inside and led them to her mother’s bedroom. She gave the officers the bag, which they took to the station house for a lab test. After the results confirmed that the bag contained cocaine, the mother was arrested and charged with illegal possession of a controlled dangerous substance.
    The mother’s attorney has filed a motion to prevent the cocaine from being admitted into evidence. The motion should be
    (A) denied, because the daughter, and husband, as co-owner, had the authority to allow the police inside the home and make the seizure.
    (B) denied, because the daughter lived with her mother and had equal control over the premises.
    (C) granted, because the daughter, as a minor, did not have apparent authority to permit the police to enter the home.
    (D) granted, because the daughter did not have apparent authority to permit the police to enter her mother’s bedroom.
A
  1. (D) Once again, it is necessary to use the third-party analysis previously stated. First and foremost, did the daughter have authority “in her own right” to allow the police to search her mother’s bedroom? The answer is no. Although the child may have had the apparent authority to permit the police to search common areas of the home (e.g., living room or kitchen) the daughter clearly did not have authority to allow the police to search her mother’s bedroom. Moreover, because the mother locked the bedroom door, she did notassume the risk that her daughter would gain entry. Therefore, choices (A) and (B) are incorrect. Choice (C) is incorrect because it is a misstatement of law.
55
Q
  1. A teenager is the star player on his high school basketball team. The high school is a public school, and the school has a policy that all student athletes are required to sign an authorization form by which they consent to undergo random drug testing. All the basketball players on the school team, except the teenager, signed the authorization.
    In the third game of the season, the team beat their archrival, and the teenager scored a record high 69 points. In the locker room following the game, the basketball coach approached the teenager and requested that he provide a urine sample for a random drug test. The teenager refused. He had smoked some marijuana the previous day and was afraid that a positive test result might jeopardize his being awarded a basketball scholarship to college.
    Thereafter, the teenager was suspended from the team for failing to comply with the coach’s request. The coach later admitted that he did not have any individualized suspicion that the teenager was under the influence of drugs.
    The teenager has filed a motion challenging the suspension and seeking reinstatement on the basketball team.
    The court will most likely rule in favor of
    (A) the school, because a urine sample does not constitute a search under the Fourth Amendment.
    (B) the school, because drug testing is not an intrusion on his Fourth Amendment right of privacy.
    (C) the student, because he didn’t consent to the drug test.
    (D) the student, because the coach admitted that he did not have any individualized suspicion that the student was under the influence of drugs.
A
55. (B) In VernoniaSchoolDistrict47Jv.Acton, 515 U.S. 646 (1995), the Court upheld the validity of drug testing of student athletes, stating that “the intrusion upon privacy was limited since the tests were taken under conditions typically encountered in public restrooms and the test results were disclosed to a limited class of school personnel who have a need to know.” The intrusion, the Court concluded, was outweighed by the legitimate government interests advanced (namely, deterring drug use by school children and preventing physical harm to drug users and other players). Choice (A) is wrong because drug testing does constitute a search (of student athletes) but, under the circumstances, it is lawful. Choice (C) is incorrect because student consent is not an issue. Choice (D) is incorrect because, based on the Vernonia decision, it is not necessary for a coach to demonstrate an individualized
suspicion to authorize a random drug test.
56
Q
  1. During spring break, a private boarding school was deserted while students and teachers were away on vacation. A guidance counselor remained on campus because he was working on a research project. After working late one night, the counselor decided to enter the room of a student from a very wealthy family. The counselor was rummaging through the student’s room looking for something valuable to steal. Under the bed, he noticed an expensive suitcase. The counselor opened the suitcase and found an express mail envelope. The counselor knew that the student’s father often sent money to his son in express mail envelopes. The counselor opened the envelope and saw that it contained a large quantity of white powder, which he suspected to be heroin. The counselor telephoned the police, and an officer was dispatched to the school. The counselor handed the officer the envelope, which he transported to the police station. At the station house, the officer opened the envelope and sent a sampling of the substance to the police lab. Tests confirmed the substance to be heroin. The police did not secure a search warrant before confiscating and opening the envelope. The student was thereafter arrested and charged with unlawful possession of a controlled dangerous substance.
    The student’s attorney has filed a motion to suppress the heroin from evidence. The motion will most likely be
    (A) granted, because the police should have secured a warrant before seizing the envelope.
    (B) granted, because the police should have secured a warrant before opening the envelope.
    (C) denied, because the search was conducted by a private party.
    (D) denied, because the counselor, as a school employee, was in loco parentis.
A
  1. (C) By now, you’ll note that the key to the Multistate is recognizing and focusing on the subtle nuances and hornbook distinctions that are being tested. Here, the search of the student’s suitcase was done by a guidance counselor who acted as a private person. The Fourth Amendment (prohibition against unreasonable searches and seizures) generally protects only against governmental conduct and not against searches by private persons. Choice (C) is, thus, the best answer. Choices (A), (B), and (D) are incorrect for the reasons stated above.
57
Q
  1. A man was under suspicion for participating in the commission of a bank robbery. A grand jury was convened to consider whether there was sufficient evidence against the man to indict him for the crime. During the grand jury hearing, the state called a police detective to testify. The detective testified that following the bank robbery, he interviewed a bystander who stated that his friend witnessed the robbery and told the bystander that the man was the person who committed the crime.
    Neither the man nor his attorney was allowed to be present during the grand jury proceeding. As a result, the man did not have the opportunity to cross- examine the detective or to object to the admissibility of his testimony. A grand jury indictment against the man was issued, and the prosecution has formally charged the man with bank robbery. While the man awaits trial, his attorney has filed an appropriate motion to dismiss, challenging the admissibility of evidence during the grand jury proceeding.
    The motion should be
    (A) denied, because the issue before the grand jury is probable cause, not guilt.
    (B) denied, because although the detective’s testimony is hearsay, rules of evidence other than privilege are not applicable to grand jury proceedings.
    (C) granted, because the grand jury hearing violated the man’s Sixth Amendment privilege of confrontation, since there was no opportunity to cross-examine the detective.
    (D) granted, because the grand jury indictment was based upon inadmissible evidence that resulted in an unwarranted prosecution.
A
  1. (A) & (B) Credit is sometimes given on the MBE for two correct answers. Here, double credit should be given for both (A) and (B). The primary function of a grand jury is to determine probable cause to prosecute, not guilt. Clearly (A) is correct. Choice (B) is equally correct because FRE 1101(d) states that in grand jury proceedings, “rules of evidence other than privilege” are inapplicable. Choices (C) and (D) are wrong because the validity of a grand jury indictment is not affected by the character of evidence considered.
58
Q
  1. One day, an 11-year-old boy came home from school with red lipstick stains on his collar. When the boy’s mother inquired about the lipstick stains, the boy said that his teacher had kissed him.
    The mother then contacted the police, who subsequently conducted an investigation and arrested the teacher, charging her with two felony counts of child molestation. In accord with state law, a preliminary hearing was scheduled. After the boy was subpoenaed to testify at the preliminary hearing, the prosecution filed a motion to exclude the teacher from the courtroom during the boy’s appearance. The motion, with supporting affidavits from a child psychologist, claimed that having the boy and the teacher in the same room would cause irreparable emotional and psychological harm to the alleged victim.
    The judge should
    (A) deny the motion, because a pretrial proceeding is an early stage of trial, and a defendant has a constitutional right of confrontation at every stage of the trial.
    (B) deny the motion, because the teacher’s exclusion from the courtroom would have prejudicial impact on receiving effective assistance of counsel.
    (C) grant the motion, because the teacher’s attorney still has an opportunity to cross-examine the witness despite the teacher’s absence from the courtroom.
    (D) grant the motion, because cross-examination at a preliminary hearing is not required by the confrontation clause of the Sixth Amendment.
A
  1. (D) All jurisdictions grant the defense a right to cross-examine those witnesses presented by the prosecution at the preliminary hearing. The right is based on local law (usually by statute or court rule). The Supreme Court has long held that cross- examination at a preliminary hearing is not required by the confrontation clause of the Sixth Amendment. LaFave, Criminal Procedure, pg. 677. The purpose of a pretrial hearing is to determine probable cause, not to obtain discovery. Thus, the magistrate has broad discretion to limit or cut off cross-examination of witnesses. Choice (C) is wrong because it indicates that the teacher’s attorney can still effectively cross-examine the witness at the preliminary hearing despite her absence. Though this may be true, there is nonetheless no constitutional right of confrontation at the preliminary hearing stage. Thus, choice (D) provides the correct rule of law. Choices (A) and (B) are incorrect for the reasons stated above.
59
Q
  1. Police received information from a reliable informant that a defendant would be extorting money from a victim. The informant gave the police explicit details about the extortion scheme: the defendant would be calling the victim from a public phone booth on a particular street corner on a certain date and time to make the extortionist demand.
    Without notif’ing the telephone company or seeking a warrant, the police installed an eavesdropping device on the public phone. At the time on the date specified by the informant, the defendant phoned the victim from the telephone booth and said, “You better pay me $5,000 by next week or I’ll kill you and your family.” The defendant provided additional details about how and where to make the payoff.
    Based on the taped conversation, the police arrested the defendant and charged him with extortion. Prior to trial, the defendant’s attorney moves to exclude the taped conversation from evidence.
    The defense motion should be
    (A) granted, because the police did not secure a warrant before installing the eavesdropping device.
    (B) granted, because the police did not obtain prior approval of the telephone company before installing the eavesdropping device.
    (C) denied, because the defendant did not have reasonable expectation of privacy.
    (D) denied, because the police had probable cause based on the information supplied by the informant.
A
  1. (A) Any form of electronic suiveillance, including wiretapping, that violates a reasonable expectation of privacy constitutes a search. A valid warrant authorizing any form of electronic surveillance, including wiretapping, must satisfy the following requirements, set forth in Berger v. New Yorlç 388 U.S. 41 (1967): (1) the warrant must describe with particularity the conversations to be overheard; (2) a showing of probable cause; (3) the wiretapping must be for a limited period of time; (4) the suspects must be named; and (5) a return must be made to the court showing what conversations were intercepted. Title Ill (under the Omnibus Crime Control and Safe Streets Act of 1968) also sets forth requirements for the issuance of interception or wiretap orders. Choice (A) is correct because a neutral magistrate must approve a warrant authorizing electronic surveillance. Choice (B) is wrong because a neutral magistrate, not the telephone company, has legal authority to issue a warrant authorizing wiretapping. Therefore, choices (C) and (D) are incorrect.
60
Q
  1. During a two-month span, there were 15 bank robberies in the city area. The robberies were carried out by a single individual who entered the banks wearing a ski mask and carrying a machine gun. The police received information from a reliable informant that the defendant was the person responsible for the robberies. Acting on this tip, the police secured a valid arrest warrant and went to the defendant’s home to apprehend him. When the officers rang the doorbell, the defendant opened the door and was placed under arrest.
    The defendant, who was wearing only underwear, asked the officers if he could put on a shirt and pants. One of the police officers asked the defendant if anyone else was present in the house. The defendant indicated that he was alone. The officer then accompanied the defendant to his bedroom to change his clothes. When the defendant asked permission to enter the closet for his pants, the officer told him that he needed to inspect the closet first. While the officer was inside the closet, he saw a ski mask that he recognized as part of the apparel worn by the bank robber. The officer confiscated the ski mask and then allowed the defendant inside the closet to change his clothes.
    While the officer and the defendant were in the bedroom, the officer’s partner decided to conduct a protective sweep of the other rooms of the dwelling to see if any accomplices were present. Upon entering a back storage room, the other officer saw a box containing a large amount of $100 bills. The officer confiscated the money, which was later determined to be part of the money stolen during one of the bank robberies.
    The defendant is subsequently prosecuted for bank robbery. The prosecution seeks to introduce the ski mask and the money found in his home into evidence. A motion by the defendant’s attorney to exclude the items will be
    (A) granted with respect to the ski mask, but denied with respect to the money.
    (B) granted with respect to the money, but denied with respect to the ski mask.
    (C) granted with respect to both the ski mask and the money.
    (D) denied with respect to both the ski mask and the money.
A
  1. (B) In Washington v. Chrisman, 455 U.S. 1 (1982), the Supreme Court held that the police may accompany the defendant into the home following his arrest so that the defendant may obtain identification, get his effects, or change clothes. It is not unreasonable underthe Fourth Amendment fora police offlcerto monitorthe movements of an arrested person, as his judgment dictates, following arrest. Therefore, the officer was justified to Inspect the cLoset before aLlowing the defendant inside to change his clothes. The seizure of the ski maskwas in “plain view” and, thus, admissible in evidence. On the contrary, the officer did not have reasonable grounds to conducta protective sweep of the premises. A protective sweep is permissible if the police have a “reasonable and articulable suspicion” that accomplices are present in the dwelling and pose a danger to those on the arrest scene. There was no justification for a protective sweep because the facts stated that “a single individual” was responsible for the bank robberies. In sum, the ski mask will be admitted into evidence and the cash excluded. Therefore, choices (A), (C), and (D) are incorrect.
61
Q
  1. A defendant was arrested and charged with possession of heroin. At the defendant’s preliminary hearing, the prosecution presented evidence that the defendant was arrested while riding in a car. The heroin was found in a briefcase containing no identification, but several papers found inside contained references to the defendant. The defendant’s attorney argued that the briefcase did not belong to the defendant, and the defendant had no idea how the heroin got inside the briefcase. The court ruled that there was insufficient probable cause to support a prosecution of the defendant on the heroin possession charge. The defendant was released from custody and charges were dropped.
    Several weeks later, a grand jury was convened to determine if the defendant should be re-indicted for heroin possession based on the same alleged incident reviewed at the preliminary hearing. The defendant was called to testif’ as a witness at the grand jury hearing. He appeared and was sworn in as a witness. The defendant proceeded to challenge the proceeding on the basis that any grand jury indictment for heroin possession would constitute a violation of the double jeopardy clause.
    The defendant’s double jeopardy challenge should be
    (A) granted, because by voluntarily appearing at the grand jury hearing, the defendant has standing to challenge the indictment.
    (B) granted, because the heroin possession charge was formally dismissed at the preliminary hearing.
    (C) denied, because there is no basis for a double jeopardy challenge.
    (D) denied, because double jeopardy rights do not attach unless there has been an acquittal or conviction.
A
  1. (C) According to LaFave, a dismissal at the preliminary hearing may be appealed or subject to “reversal” by a grand jury. The dismissal occurs “before jeopardy has attached and the 5th Amendment does not barinitiation of a new prosecution for the same offense.”Although choices (C) and (D) are both arguably correct, choice(D) is less preferred because it states that double jeopardy does not attach unless there has been an acquittal or conviction. This statement is not entirely correct because double jeopardy attaches after there has been a “final judgment.” In United States v.Jorn, 400 U.S. 470 (1971), the Supreme Court held that “final judgments” are not limited to acquittals and convictions and even extend to situations involving prosecutorial or judicial overreaching. In jorn, double jeopardy barred re-prosecution where a mistrial was declared due to prosecutorial misconduct. As such, choice (C) is a better answer. Choices (A) and (B) are incorrect for the reasons stated above.
62
Q
  1. A defendant, an indigent, was arrested and charged with attempted murder. The defendant, who had a history of psychiatric problems, had been diagnosed as being paranoid schizophrenic. The defendant was represented by a court-appointed attorney. In trying to prepare for trial, the attorney met with the defendant on numerous occasions. The attorney, however, found it difficult to communicate with the defendant, who, due to his paranoia, distrusted the attorney and believed he was working with the prosecution to convict him. The defendant was subsequently convicted.
    After the defendant was sentenced, the attorney filed an appeal seeking to overturn the conviction and have a new trial ordered. The appeal, in part, challenged the constitutionality of a state statute that placed the burden of proof on the defendant by requiring him to prove mental incompetency by clear and convincing evidence. In addition, the appeal claimed that the attorney was unable to provide adequate representation because there was a failure of communication between the defendant and his attorney.
    The appeal will most likely be
    (A) granted, because the prosecution has the burden to prove the defendant’s competency by a preponderance of evidence.
    (B) granted, because the defendant has the burden to prove mental incompetency by a preponderance of the evidence, not by clear and convincing evidence.
    (C) denied, because it is constitutionally permissible to place the burden on the defendant to prove mental incompetency by clear and convincing evidence.
    (D) denied, because the defendant cannot base an appeal on ineffective assistance of counsel due to his own failure to communicate with his lawyer.
A
  1. (A) This question typifies the fine-line distinctions and nuances commonly tested on the Multistate. It is important to distinguish between defendant’s competency to stand trial and the insanity defense. Insanity is a defense to a criminal charge; a defendant acquitted by reason of insanity may not be retried and convicted, although she may be hospitalized under certain circumstances. On the other hand, competency to stand trial depends on a defendant’s mental condition at the time of trial, unlike insanity, which turns on a defendant’s mental state at the time of the crime. Competency is not a defense, but rather a bar, to trial. According to LaFave, “the weight of authority suggests that the prosecution bears the burden of proof of competency once the issue has been raised.” As a consequence, choices (B) and (C) are wrong because the burden is being placed on the defendant rather than the prosecution. Although the preponderance of the evidence standard is usually utilized, some states even require the prosecution to prove competency beyond a reasonable doubt. Conversely, the majority of states place the burden of proof on the defendant to present evidence of affirmative defenses (such as insanity and self-defense) by a preponderance of the evidence. Finally, choice (D) is incorrect because it is a misstatement of law.
63
Q
  1. Defendant was convicted of armed robbery. On appeal, Defendant moves for a new trial, claiming that his Sixth Amendment rights were violated because he received ineffective assistance of counsel at trial.
    Which of the following facts, if proven, would provide the best grounds to substantiate Defendant’s ineffectiveness of counsel claim?
    (A) The trial attorney lied to Defendant about her criminal trial experience.
    (B) The trial attorney failed to interview an alibi witness.
    (C) The trial attorney did not consult Defendant about her trial strategy.
    (D) The trial attorney did not believe in Defendant’s innocence.
A
  1. (B) The benchmark for judging any claim of ineffective assistance of counsel must be whether the attorney’s conduct so undermines the proper functioning of the adversarial process that the trial cannot be relied on as having produced a just result. See Strickland v. Washington, 466 U.S. 668 (1984). The objective standard for measuring attorney performance is that of “reasonably effective assistance.” The defendant has the burden to show prejudice affecting the outcome. Choice (C) is wrong because it is the attorney, not the client, who makes decisions regarding tactics and trial strategy. Choice (D) is incorrect, since an attorney need not believe in her client’s innocence in order to provide effective and zealous representation. Likewise, choice (A) is incorrect because it is not the lack of experience, but rather lack of diligence (e.g., failing to file a timely appeal or falling asleep during the trial) that may constitute ineffective assistance of counsel. Thus, choice (B) is the best answer because the Strickland test for measuring prejudice is whether there is a “reasonable probability that, but for counsel’s unprofessional errors, the result of the proceedings would be different.” Failure to interview an alibi witness certainly would impact the outcome of the case, since it relates to the legitimacy of Defendant’s defense. Obviously, if the alibi witness was credible, it would substantiate Defendant’s claim of innocence.
64
Q
  1. A father and a son were indicted and charged with bank robbery. They allegedly participated together in robbing a bank in their hometown. After the son was arrested and given Miranda warnings, he confessed and admitted that he and his father took part in the bank robbery.
    The prosecution decided to charge the father and son in the same indictment and prosecute them together in a single trial. At trial, the son took the stand and recanted his earlier confession by denying participation in the robbery.
    The prosecution now seeks to introduce into evidence the son’s earlier confession that was videotaped by the police. The son and the father both object to its admissibility. The confession should be
    (A) admitted against the son only.
    (B) admitted against both the son and the father, because they were engaged in a conspiracy.
    (C) not admitted against the son or the father, because the confession is hearsay not within any recognized exception.
    (D) not admitted against the son or the father, because the son recanted the confession.
A
  1. (B) Students must take a very systematic one-step-at-a-time approach to this question. Let’s first examine the confession made by the son. There are four ways to attack the admissibility of a statement or confession: (1) Miranda approach; (2) right to counsel approach; (3) voluntariness approach; or (4) the fruits of the poisonous tree doctrine. The son validly waived his Miranda rights when he confessed after being given his Miranda warnings. The confession was voluntarily made and not a fruit of a prior Fourth Amendment illegality. Therefore, the confession will be admissible. The videotaped confession is a written assertion (i.e., a statement of a party being offered against him by the prosecution). The videotape will be admissible substantively as an admission. Now for the second step: Is the confession admissible against the son only or against his father as well? In Bruton v. United States, 391 U.S. 123 (1968), the Court concluded that merely giving a precautionary instruction to the jury to consider the confession as evidence only against the defendant who made it impermissibly infringes the right of confrontation of other co-defendants. However, the Bruton situation, based on the confrontation clause issue, arises only where the confessing defendant refuses to take the stand and testify at trial. In this question, the son does take the stand and recants his confession, so Bruton is not controlling. In Nelson v. O’Neil, 402 U.S. 622 (1971), the Court concluded that the opportunity to cross-examine the declarant (son) at trial, at least where he repudiates the confession, sufficiently protects the non-confessing defendant’s right of confrontation. Therefore, the confession will be admitted against both the son and his father. Choice (B) is the correct answer. Choices (A), (C), and (D) are incorrect for the reasons stated above.
65
Q
  1. Police received a tip from a reliable informant that a man was dealing cocaine from his home. Officers then submitted to a magistrate an affidavit setting forth circumstances showing probable cause for the issuance of a search warrant. The magistrate issued a search warrant for narcotics at the man’s address.
    Police officers went to the man’s home and rang the door bell. When no one answered, the police broke in and entered the dwelling. They searched the rooms on the first floor and found no incriminating evidence, then walked outside and noticed a separate entrance leading up to the second floor of the dwelling. They ascended the stairway and entered an upstairs bedroom. Inside the room the officers found a stash of cocaine lying on a dresser. As the officers were continuing their search of the bedroom, the defendant entered the room. The police asked the defendant if he was the man. The defendant identified himself and told the police that they were in his bedroom and he occupied the second floor of the dwelling. The defendant explained that the man resided on the first floor and his living quarters were separate from the defendant’s. Nonetheless, the defendant was placed under arrest and charged with possession of cocaine.
    The defendant has filed a motion to exclude the cocaine from being admitted into evidence. The defendant’s motion should be
    (A) granted, because the warrant did not name the defendant.
    (B) granted, because the warrant did not state with specificity the area to be searched.
    (C) denied, because the police reasonably believed they were searching the man’s residence when they entered the defendant’s living quarters.
    (D) denied, because the warrant was valid and issued with probable cause.
A
  1. (C) The Fourth Amendment provides that no warrants shall issue except those “particularly describing the place to be searched.” Absolute perfection in description is not required; it “is enough if the description is such that the officer with a search warrant can, with reasonable effort, ascertain and identify the place intended.” See Steele v. United States, 267 U.S. 498 (1925). A special problem deaLs with a search warrant for an apartment house or other multiple-occupancy building. The general rule is that a search warrant will usually be held invalid if it fails to describe the particular sub-unit to be searched with sufficient definiteness to preclude a search of one or more sub-units indiscriminately. This means that, in the absence of a probab’e cause showing as to all the living units so as to justify a search of them all, a search warrant directed at a multiple-occupancy structure will ordinarily be held invalid if it describes the premises only by street number or other identification common to all the sub-units located within the structure. However, LaFave points out that an exception to that rule is if the building in question, from its outward appearance, would be taken to be a single-occupancy structure and neither the affiant other investigating officers nor the executing officers knew, or had reason to know, of the structure’s actual multiple-occupancy character until execution of the warrant was under way, then the warrant is not defective for failure to specify a sub-unit within the named building. Based on this exception, choice (C) is correct because the multiple-occupancy character of the structure was not known, and the police acted reasonably in the execution of the warrant. Therefore, choices (A) and (B) are incorrect. Choice (D) is incorrect because it is not as specific as to the issue presented in the question as is choice (C).
66
Q
  1. A defendant parked his car illegally in a handicapped parking space while he went into a store to get a cup of coffee. The defendant was inside the store waiting for his coffee when a member of the city’s parking enforcement department drove by the vehicle. Seeing that there was no handicapped sticker on the car, the parking official stopped and started ticketing the vehicle. When the defendant came outside, he got into an argument with the parking official. After the defendant became verbally abusive, the parking official told him to desist or he was going to impound the vehicle. After the defendant continued his badgering, the parking official told him that he was impounding the vehicle and called the station house for officer assistance.
    A few minutes later, a police officer arrived on the scene. He asked the defendant for identification and then proceeded to frisk him for weapons. The officer found a clear vial containing heroin in the defendant’s back pocket. He then placed the defendant under arrest.
    Thereupon, the parking official went into the defendant’s car and began an inventory search. In this jurisdiction, parking enforcement personnel have statutory authority to conduct inventory searches of impounded vehicles. The parking official did not find anything of interest. After the police officer placed the defendant in the patrol car, the officer asked the parking official if he could search the vehicle, to which the parking official responded yes. The officer looked under the floor mat and found a bag containing marijuana. The defendant was taken to the station house and charged with unlawful possession of heroin and marijuana.
    Prior to trial, the defense filed a motion to exclude the heroin and marijuana from being admitted into evidence. The motion will be granted with respect to
    (A) the heroin only.
    (B) the marijuana only.
    (C) both the heroin and the marijuana.
    (D) neither the heroin nor the marijuana.
A
  1. (C) Here’s a question dealing with search and seizure of impounded vehicles. First, the police may conduct an inventory of the contents of vehicles they have taken into their custody in order to protect the vehicle and the property in it and to safeguard the police from claims of lost possessions. In South Dakota v. Opperman, 428 u.s. 364 (1976), a defendant’s illegally parked car was towed to the city impound lot where an officer, observing articles of personal property in the car, proceeded to inventory it, and found a bag of marijuana in the unlocked glove compartment. The Court upheld the validity of the search and ruled that the conduct of the police was not unreasonable. However, most courts hold that an inventory search of a lawfully impounded vehicle must be necessary in light of standardized criteria, such as protecting the owner’s property. Impoundment may not be used as a pretext for a search for evidence of a crime or a subterfuge for an exploratory search. See Opperman, Colorado v. Bert/ne, 479 U.S. 367 (1987). In this question, the initial impoundment by the parking official was not necessary just because the defendant was verbally abusive. The vehicle could have been moved out of the handicapped space and was not creating a traffic hazard. The frisk that followed was clearly unlawful under a Terryv. Ohio standard, since the police officer had no reasonable suspicion to believe the defendant was armed or dangerous. Therefore, the heroin found during the frisk was unlawfully seized and should be excluded from evidence. Regarding the second inventory search by the police officer, it was not a routine, standardized search to protect the owner’s property. The police officer was seeking to gather criminal evidence. The marijuana found was not discovered incident to a lawful arrest, but rather was the fruit of an unlawful Terry frisk followed by an illegal arrest followed by an improper inventory search. Choice (C) is correct. Choices (A), (B), and (D) are incorrect for the reasons stated above.
67
Q
  1. A defendant owned a large parcel of farmland located in a rural section of a state. He conducted a small farming operation on the property and grew corn, broccoli, and other vegetables, which he sold to local produce companies. One day, the county police received an anonymous tip that the defendant was growing marijuana on his farm. Acting on this information, in an unmarked vehicle, two undercover police officers then drove to the defendant’s farm. They climbed over a chain-link fence and walked around the farm. While surveying the terrain, they came upon a small marijuana patch. The officers then noticed fresh footprints that led from the marijuana patch to a nearby cabin.
    Believing that marijuana was being stored in the cabin, the police officers decided to immediately secure a search warrant. After obtaining the warrant, they returned to the defendant’s farm and entered the cabin. Inside the cabin, the police found a large amount of marijuana that had been recently harvested. The defendant was then arrested and charged with unlawful possession of marijuana.
    The defendant files a pretrial motion to suppress the marijuana as evidence on grounds of an illegal search and seizure. The motion should be
    (A) denied, because the officers secured a warrant before entering the cabin despite the fact that their earlier actions may have been unlawful.
    (B) denied, because the warrant was validly issued and based upon information lawfully obtained.
    (C) granted, because the marijuana was the fruit of an illegal search and seizure, since the police did not have probable cause to conduct their investigation.
    (D) granted, because the police were unlawfully on the defendant’s property when the marijuana was initially discovered.
A
  1. (B) According to the “open fields” doctrine, any unoccupied or undeveloped area outside of the curtilage (such as wooded areas, desert, vacant lots in urban areas, open beaches, reservoirs, and open waters, even if fenced or posted with no trespassing signs) is not afforded Fourth Amendment protection. Choice (A) is wrong because the conduct of the police was not unlawful. Choice (C) is incorrect because the marijuana was not the “fruit” of an illegal search and seizure. Choice (D) is incorrect because the police could enter the defendant’s farm without a warrant, since it was an “open field.” The “open fields” doctrine was reaffirmed by the U.S. Supreme Court, under a very similar set of facts as we have present in our question, in 01/very. United States, 466 U.S. 170 (1984).
68
Q
  1. During a defendant’s trial for driving while intoxicated, the prosecutor called a toxicologist to testif’ as an expert witness. The toxicologist testified that the defendant’s blood alcohol level, registered after his arrest, was 0.14%. At the close of the toxicologist’s testimony, the trial judge, over defense objections, instructed the jury that, in accordance with state law, a person with a blood alcohol level greater than 0.10% is presumed to be intoxicated. No other instruction relating to that presumption was given. A relevant statute in this jurisdiction makes driving while intoxicated a felony and defines intoxication “as a state of severely impaired mental and physical capacity caused by the ingestion of drugs or alcohol.”
    The defendant was subsequently convicted of driving while intoxicated. On appeal, his conviction should be
    (A) affirmed, because the defendant had an opportunity to rebut the presumption.
    (B) affirmed, because the judge’s instruction merely permitted the jury to reach a conclusion that they could have decided upon without the presumption.
    (C) reversed, because the jury might determine that the instruction in effect relieves the prosecutor of the burden of proof beyond a reasonable doubt.
    (D) reversed, because such an instruction may be considered an indication of guilt and, therefore, violative of the defendant’s privilege against self-incrimination.
A
  1. (C) Revised Uniform Rule of Evidence 303 covers presumptions in criminal cases. Under section 303(b), “If a presumed fact establishes guilt … the court may submit the question of guilt … to the jury, but only if a reasonable juror on the evidence as a whole, including the evidence of the basic facts could find guilt or the presumed fact beyond a reasonable doubt.” Moreover, under Uniform Rule 303(c), “… if the presumed fact established guilt … the court shall instruct the jury that its existence, or all the evidence, must be proved beyond a reasonable doubt.” In this question, the facts state, “No other instruction related to the presumption was given.” Since the presumption, namely, intoxication, based upon the blood alcohol level greater than 0.10% indicated guilt of the defendant, the judge should have given a further jury instruction to require proof beyond a reasonable doubt. The judge did not do so. Therefore, on appeal, the defendant’s conviction should be reversed because the jury might determine that the instruction, in effect, relieves the prosecutor of the burden of proof beyond a reasonable doubt. Choice (C) is correct. Therefore, choices (A), (B), and (D) are incorrect.
69
Q
  1. A wife was divorced from her husband. The wife and the husband had a daughter who was 12 years of age. The daughter lived with the wife, but the husband frequently visited his daughter at her house. One morning, the husband came to his ex-wife’s home to see his daughter. He and his daughter were outside on the patio chatting when the wife noticed her ex-husband’s briefcase lying on the kitchen floor. The husband had entered the house carrying his briefcase, which he placed down before going out to see his daughter on the patio.
    Seeing the briefcase, the wife decided to open it. Inside she found a large envelope. She decided to keep it, thinking it contained money. Later that night after her ex-husband left, she opened the envelope and found a large amount of cocaine.
    The wife called the police who came and confiscated the drugs. The husband was subsequently charged with possession of cocaine.
    The husband has filed a motion to exclude the cocaine from being introduced into evidence. His motion should be
    (A) granted, because the seizure violated the husband’s right of privacy.
    (B) granted, because the police failed to secure a search warrant before the seizure of the cocaine.
    (C) denied, because the initial seizure was made by a private individual.
    (D) denied, because the husband left the briefcase in the kitchen in plain view.
A
  1. (C) The Fourth Amendment generally protects only against “governmental conduct” and not against searches by private persons. Searches by private individuals who are not acting as agents of government officials do not implicate the Fourth Amendment. An individual can claim Fourth Amendment protection only when “governmental conduct” infringes upon society’s “reasonable expectations of privacy,” and further infringes upon the individual’s “legitimate expectations of privacy.” Here, the cocaine was initially “seized” by the wife, who was not a government agent. She, in turn, gave the cocaine to the police. In all likelihood, since the police received the cocaine from the wife, they would not be required to secure a warrant. Therefore, choices (A), (B), (D) are incorrect.
70
Q
  1. Under which of the following situations would imposition of the death penalty most likely be justified in light of constitutional considerations?
    (A) A defendant, a convict in prison, stabbed a prison guard to death and by statute is mandated to receive the death penalty.
    (B) A defendant kidnapped and sexually assaulted a 12-year-old girl for a week before the defendant was arrested.
    (C) A defendant raped his victim and then savagely beat her with a baseball bat, causing her to suffer permanent brain damage.
    (D) A defendant shot and killed a police officer during an attempted bank robbery.
A
  1. (D) Choices (B) and (C) are incorrect because in Cokerv. Georgia, 433 U.S. 584 (1977), the Supreme Court concluded that “death is indeed a disproportionate penalty for the crime of raping an adult woman.” Coker is, to be sure, a most significant decision because it “announced a principle of morality in law, namely, society may not take the life of a defendant who has not taken the life of his victim.” Interestingly, choice (A) is wrong because in Sumner v. Shuman, 483 U.S.66 (1987), a statute that mandates the death penalty for a particular category of defendants (prison inmates) is unconstitutional because there is no meaningful opportunity for considering mitigating factors. Thus, by process of elimination, choice (D) is the best of the available answers.